Haematological Pathology Flashcards

1
Q

What is Haemopoiesis?

A
  • Physiological developmental process that gives rise to cellular components of blood (from stem cells)
  • A single multipotent haemopoietic stem cell can divide & differentiate to form diff cell lineages which will populate the blood
  • (Haemopoiesis= Haematopiesis= hematopoiesis)
  • Stem cells give rise to sufficient n.o.s of committed haemopoetic progeniators to maintain bloods cellular contents throughout individuals lifespan.
  • Differentiation; myelopoiesis, lymphopoiesis.
  • Mature blood cells have a finite life e.g. an erythrocytes (RBC)- 120 days, neutrophils (WBC)- 6-10 hours in bloodstream.
  • Serial divisions & prolifs that end in specialised blood cells
How well did you know this?
1
Not at all
2
3
4
5
Perfectly
2
Q

What are the properties of Haemopoietic Stem Cells?

A
  • Self renewal
  • High proliferative potential
  • Differentiation potential for all lineages of cells in blood stream
  • Long term activity throughout individuals lifespan
  • Experimental proof from stem cells transplanted from 1 mouse to another over several generations- stem cells keep working.
How well did you know this?
1
Not at all
2
3
4
5
Perfectly
3
Q

What are the origins of Haemopoiesis?

A
  • Fertilised ovum= replicates= morula embeds in wall= trophoblast; areas where hemopoieses can start.
  • Yolk sac
How well did you know this?
1
Not at all
2
3
4
5
Perfectly
4
Q

What is the timescale of Haemopioesis in the embryo?

A

Haemopioesis strats in Aorto-Gonado- Mesonephros
• Day 27: Haemopioesis starts at in aorta gonad mesonephros region.
• Day 35: Expands rapidly
• Day 40: then disappear at day 40- ‘disappearance’ correlates with haematopoietic stem cells to foetal liver which becomes subsequent type of haemopioesis
• Area in base of aorta- is the AGM where primitive stem cell they then perculate through aorta wall to the liver.

How well did you know this?
1
Not at all
2
3
4
5
Perfectly
5
Q

What is the function of Blood Cells?

A
Blood Cell Function 
•	Oxygen transport 
•	Coagulation (haemostasis)
•	Immune response to infection 
•	Immune response to abnormal cells (screens out senescent & malignant cells)
How well did you know this?
1
Not at all
2
3
4
5
Perfectly
6
Q

What are erythrocytes? What happens if they are reduced? What happens when they are increased?

A
RED BLOOD CELLS 
•Bi-concave discs, 7.5 µM diameter
•Lifespan in blood: 120 days (3mnths)
•Contain Hb
•333,200 x 106 red cells 
•Reduced RBCs= ANAEMIA
•Raised RBCs= POLYCYTHEMIA
(Relative polycythaemia- when plasma vol reduced so looks like RBCs high)
How well did you know this?
1
Not at all
2
3
4
5
Perfectly
7
Q

What are some common leukocytes? What is the function of leukocytes?

A
WHITE BLOOD CELLS 
•	Functions: immunity & host defence
•	Granulocytes- have cytoplasmic granules;
o	Neutrophils
o	Eosinophils
o	Basophils 
•	Monocytes
•	Lymphocytes
How well did you know this?
1
Not at all
2
3
4
5
Perfectly
8
Q

What are neutrophils?

A
  • Phagocytes
  • Polylobed nuclei (n.o. of lobes can indicate anaemia)
  • Most common EBC in adult blood (~ 10x109 per litre)
How well did you know this?
1
Not at all
2
3
4
5
Perfectly
9
Q

What is neutrophilia?

A

Increased numbers of Neutrophils= neutrophilia e.g. bacterial infec/ inflamm

How well did you know this?
1
Not at all
2
3
4
5
Perfectly
10
Q

What is neutropenia?

A

decreased numbers of Neutrophils = neutropenia e.g. side effect of drug (steroids act on neutrophils on endothelial wall = cause neutrophils to leave wall & float out into blood (demargination))

How well did you know this?
1
Not at all
2
3
4
5
Perfectly
11
Q

What is eosinophilia ?

A

Increased numbers of Eosinophils = eosinophilia e.g. parasitic infec, allergies (e.g. asthmatic)

How well did you know this?
1
Not at all
2
3
4
5
Perfectly
12
Q

What is basophilia ?

A

Increased numbers of basophils: e.g. chronic myeloid leukaemia (member of myloprolif neoplasms)

How well did you know this?
1
Not at all
2
3
4
5
Perfectly
13
Q

What are monocytes?

A
  • Phagocytic &; antigen presenting cells

* Migrate to tissues- then i.d. as ‘macrophages’ or ‘histiocytes’ e.g. kupffer cells (liver), Langerhans cells (skin).

How well did you know this?
1
Not at all
2
3
4
5
Perfectly
14
Q

What is monocytosis ?

A

•Increased numbers of monocytes= MONOCYTOSIS e.g. tuberculosis

How well did you know this?
1
Not at all
2
3
4
5
Perfectly
15
Q

What is lymphocytosis?

A

•Increased numbers of Lymphocytes: lymphocytosis e.g.
oAtypical lymphocytes of glandular fever (infectious mononucleosis)
oChronic lymphocytic leukaemia

How well did you know this?
1
Not at all
2
3
4
5
Perfectly
16
Q

What is lymphopenia?

A

•Decreased numbers of Lymphocytes: - lymphopenia e.g.

o Post bone marrow transplant (lymphocyte cells one of last to regenerate after this

How well did you know this?
1
Not at all
2
3
4
5
Perfectly
17
Q

What are Natural Killers cells?

A

Lymphocytes
• Innate immune system
• Larger granular lymphocytes
• Recognise ‘non-self’ (cells, viruses)- go round body looking for strange/ infected cells

How well did you know this?
1
Not at all
2
3
4
5
Perfectly
18
Q

What are T-lymphocytes?

A
Lymphocytes
•	Adaptive immune system (specific response to attack)
•	Multiple sub-types e.g. CD4 antigen 
•	Rearrange T-cell antigen recep 
•	Cell-mediated immunity 
•	Target- specific cytotoxicity 
•	Interact with B cells &; macrophages
•	Regulate immune response
How well did you know this?
1
Not at all
2
3
4
5
Perfectly
19
Q

What are B-lymphocytes?

A

Lymphocytes
• Adaptive immune system
• Rearrange immunoglobulin genes to enable antigen specific antibody production (germline configuration edited)
• Humoral immunity- immunoglobulins
• Increased numbers of plasma cells (make antibodies)- PLASMOCYTOSIS (can be benign/ malignant) e.g. infec, myeloma

How well did you know this?
1
Not at all
2
3
4
5
Perfectly
20
Q

What is a Full Blood Count (FBC)?

A

• Hb conc
• Red cell parameters;
o MCV (mean cell vol- size of individual RBC)
o MCH (mean cell Hb- how much colour in RBC)
• White cell count (WCC)
• Platelet count

Interpreting Full Blood Count
• Be alert to technical probs e.g. thrombocytopenia (may be real/ artefact)
• If in doubt if its an actual prob- repeat taking blood test
• Abnormal results; flagged by lab, may trigger additional tests e.g. blood film
• Serious urgent abnormalities- lab staff will alert on-call docs e.g. new leukaemia

How well did you know this?
1
Not at all
2
3
4
5
Perfectly
21
Q

What is a Coagulation Screen?

A

• Tests measure time taken for clot to form when plasma is mixed with specified reagents
• Can analyse diff parts of coag cascade;
o Prothrombin time (repeat this when patient on warfarin)
o Activated Partial Thromboplastin Time (unrationed heparin)
o Thrombin time (overall view of coag system in patient)

How well did you know this?
1
Not at all
2
3
4
5
Perfectly
22
Q

How do you take a blood sample?

A

• Accurate full blood count (FBC) & correct blood film interpretation;
o Appropriate sample from patient
o Collected into EDTA (anticoagulant) anticoagulated blood;
- Mixed well (shake gently)
- [K2EDTA] = 1.5 - 2.2 mg ml-1
- Blood should be filled to the line on tube
o Samples to lab promptly- as EDTA artefact can affect results.

How well did you know this?
1
Not at all
2
3
4
5
Perfectly
23
Q

Where is bone marrow obtained from?

A
  • Local anaesthetic
  • Liquid marrow aspirated from post. Iliac crest of pelvis

use tool called trephine to make a hole

How well did you know this?
1
Not at all
2
3
4
5
Perfectly
24
Q

What is a reference range?

A

Establishing a Reference Range
• Define reference pop- should be relevant to test pop
• Consider if diff reference ranges needed for; adults vs children, men vs women etc.
• Determine expected range of interindividual variation

Reference Range
• Set of values for a given test that incorporates 95% of normal pop
• 95% is arbitrary convention- determined by collecting data from vast n.o. of lab tests
• More people sample- less likely will be distorted

95% Results Fall Within Reference Range = normal

  • Differentiates between healthy & ill.
  • False Neg &; False Positive= need to figure out how good is test.
How well did you know this?
1
Not at all
2
3
4
5
Perfectly
25
Q

What is Sensitivity?

A
  • Proportion of ABNORMAL results correctly classified
  • Expresses ability to detect a true abnormality
  • Sensitivity= TP/ (TP+FN)

TP-true positive
FN- false negative

How well did you know this?
1
Not at all
2
3
4
5
Perfectly
26
Q

What is Specificity?

A
  • Proportion of NORMAL results correctly classified by test
  • Expresses ability to exclude abnormal result in healthy person
  • Specificity= TN/ (TN+FP)
  • (True normal people divided by all the normal people)

TN-true negative
FP- false positive

How well did you know this?
1
Not at all
2
3
4
5
Perfectly
27
Q

What is the importance of looking at clinical data as well as Normal Ranges?

A

• Know your patient- give clinical details
• A FBC may fall outside ‘normal range’ BUT be appropriate for the given clinical situation e.g. abnormal lymphocyte count;
o Post-splenectomy (spleen out) mild lymphocytosis
o 3 months post-bone marrow transplant lymphopenia (so lymphocyte levels should be low at that point)

How well did you know this?
1
Not at all
2
3
4
5
Perfectly
28
Q

What is Microcytic Hypochromic Anaemia ?

A
  • MCV <80 fl & MCH <27 pg
  • Small RBCs made due to not enough Hb
  • Iron deficiency
  • Thalassaemia
  • Anaemia of chronic disease (some)
  • Lead poisoning
  • Sideroblastic anaemia (some cases)

MICROCYTIC- small red cells, iron deficiency, Hb disorders (e.g. sickle cell, sometimes chronic disease)
• Anaemic patient in front of you- look at MCV, have their drug history- lead you down one of them

How well did you know this?
1
Not at all
2
3
4
5
Perfectly
29
Q

What is Normocytic Normochromic Anaemia ?

A
  • MCV 80-95 fl (normal) & MCH ≥ 27 pg (normal)
  • Failure to make enough RBCs (e.g. if get rheumatoid arthritis- turns vol of bone marrow down so less RBCs produced, but RBCs themselves are normal)
  • Many haemolytic anaemias
  • Anaemia of chronic disease (some cases)
  • After acute blood loss
  • Renal disease
  • Mixed deficiencies
  • Bone marrow failure (e.g. post-chemo, infiltration by carcinoma)

• NORMOCYTIC- normal red cells, anaemia of chronic disease/ inflammatory

How well did you know this?
1
Not at all
2
3
4
5
Perfectly
30
Q

What is Macrocytic Anaemia?

A
  • MCV >95 fl
  • Megaloblastic (defective DNA synthesis = abnormally large erythroblasts): vit B12 or folate deficiency
  • Non-megaloblastic: alcohol, liver disease, myelodysplasia, aplastic anaemia etc
How well did you know this?
1
Not at all
2
3
4
5
Perfectly
31
Q

What is Iron Deficiency Anaemia ?

A
  • Small pale red cells (low MCV &; MCH)
  • Variable size & shape- long thin ‘pencil’ cells.
  • Common in women esp. if have multiple preg, don’t eat meat & menstruating
How well did you know this?
1
Not at all
2
3
4
5
Perfectly
32
Q

What is Vit B12 Deficiency?

A
  • Hyper segmented neutrophils &; oval mastocytes
  • Big RBCs.
  • Neutrophils hypersegmented
How well did you know this?
1
Not at all
2
3
4
5
Perfectly
33
Q

What is Sickle Cell Anaemia?

A
  • Hypoxia = red cell distorted = sickle cells.

* Sickle cells crisis = bone pain (medical emergency!)

How well did you know this?
1
Not at all
2
3
4
5
Perfectly
34
Q

What cellular changes occur in Erythropoiesis ?

A
  • Nucleus changes; first open, then starts to shut down becomes darker, then extruded from RBC precursor (nucleus seen last time in reticulocyte- sometimes seen in bloodstream).
  • Megaloblastic change- nucleus stays open longer (?)
How well did you know this?
1
Not at all
2
3
4
5
Perfectly
35
Q

What are the causes of Macrocytic Anaemia ?

A
Cause of Macrocytic Anaemia 
• Megaloblastic anaemia (i.e. with megaloblastic erythropoiesis)- e.g.;
o	vit B12
o	folic acid deficiency
o	myelodysplastic syndrome
• Macrocytic anaemia with normoblastic erythropoiesis e.g. 
o	liver disease
o	alcohol
o	hypothyroidism
o	myelodysplastic syndrome 
• ‘Stress’ haemopoiesis e.g.
o	Haemolytic anaemia
o	Recovery from blood loss (macrocytes reflect high reticulocyte count- reticulocytes make machine mistakenly thinks MCV high )
How well did you know this?
1
Not at all
2
3
4
5
Perfectly
36
Q

What is Myelodysplastic Syndrome?

A

• Myeloid lineage disorder; red cells, platelets, white cells- any of these can be reduced
• How you are affected depends on which cytopenia you have (which cell type is reduced)
• Typically disease of mid-life &; older people, occasionally in young people
• May be complication of chemotherapy/ radiotherapy treatment
• Manifestations vary;
o Chronic anaemia (survival several yrs)
o Aggressive disease terminating in acute myeloid leukaemia- abnormal chromosome mutations in cells in marrow, lost of circulating blasts (primitive cells).
• Prediction of outcome in individual patients; Revised International Prognostic Scoring System (IPSS-R) for Myelodysplastic Syndromes Risk Assessment Calculator

How well did you know this?
1
Not at all
2
3
4
5
Perfectly
37
Q

What are the characteristics of Myelodysplastic Syndrome?

A
  • Mutated stem cell makes myeloid clone of abnormal cells- replaces normal haemopoiesis
  • Hyperproliferative, lots of cells in marrow but die prematurely
  • Abnormal cells; morphological abnormal maturation, often die before leaving bone marrow
  • Cellular marrow (ususally) but peripheral blood cytopenias
How well did you know this?
1
Not at all
2
3
4
5
Perfectly
38
Q

What are the lab features of Myelodysplastic Syndrome?

A
• Typical blood results;
o	Anaemia
o	Neutropenia 
o	Thrombocytopenia (single cytopenias or as a pancytopenia)
• Symptoms 
o	Anaemia- fatigue, dyspnoea 
o	Neutropenia- infec
• Thrombocytopenia- bruising &amp; bleeding 

Bone Marrow in Myelodysplasia (MDS) is Cellular - Irregular shaped nuclei &cytoplasm

How well did you know this?
1
Not at all
2
3
4
5
Perfectly
39
Q

What are the subtypes of Myelodysplastic Syndrome?

A
  • MDS has several subtypes- may be recognised morphologically/ genetically (chromosomes)
  • e.g. Refractory Anaemia, Refractory anaemia with excess blasts, MDS 5Q-syndrome
  • MDS 5Q syndrome- chromosome 5 partail loss of q arm (long arm as Q longer than the letter P)
How well did you know this?
1
Not at all
2
3
4
5
Perfectly
40
Q

What is the therapy for Myelodysplastic Syndrome?

A

•SUPPORTIVE;
o Red cell infusions (iron chelation for iron overload)
o Erythropoietin hormone (try to increase amount of RBCs they make)
o Platelet transfusions
•ACTIVE THERAPY- DISEASE SPECIFIC
o Lenalidomide for 5q- syndrome
o Azacytidine (if not fit for more intensive treatment e.g. transplant)
o Trial of immunosuppression in minority of patients
o Chemotherapy & bone marrow transplant (younger fitter patients)

How well did you know this?
1
Not at all
2
3
4
5
Perfectly
41
Q

What is Leucodepletion?

A

•Leucodepletion- whole blood filtered before further processing to remove white cells (reduce complications from WBCs & reduce transmission of creutzfeld Jacob disease)

How well did you know this?
1
Not at all
2
3
4
5
Perfectly
42
Q

What happens to blood after donation?

A
  • Blood given by donor
  • Leucodepletion- whole blood filtered before further processing to remove white cells (reduce complications from WBCs & reduce transmission of creutzfeld Jacob disease)
  • Blood then centrifuged and separated into 3 components; RBCs, platelets &plasma
  • Plasma either frozen (2 ways) or fractionated to produce plasma derived components which contain specific products
How well did you know this?
1
Not at all
2
3
4
5
Perfectly
43
Q

What are the Indications of Transfusion of RBC?

A

• 1 unit RBC;
o Usual transmission time: 1.30- 3 hrs
o 4 hr limit from removal from cold storage to end of transfusion
o Use blood warmer for rapid transfusion
• Stored at 4 degrees celcius for up to 35 days collection
• (Refer to Trust Blood Transfusion Policy)
• Most of plasma usually removed- leave ‘concentrated red cells’ &; replaced by solution of electrolytes, glycose & adenine (solution keeps RBCs healthy during storage)

How well did you know this?
1
Not at all
2
3
4
5
Perfectly
44
Q

What are the 2 stages in Pre-Transfusion Lab Testing for RBCs ):

A

(don’t need to do them for platelets/ plasma)

  1. Tests on Patient- ‘Group &; Screen’
    • Determination of ABO & Rh(D) group (of RBCs of patients)
    • Patients plasma ‘screened’ for antibodies against other clinically signif blood group antigens (other than ABO group);
    o If -ve- no further testing
    o If +ve- identify antibody; test patients plasma against panel of RBCs containing all clinically signif blood groups antigens (use antiglobulin test)
  2. Compatibility Testing- ‘Crossmatching’
    • Donor red cells with same ABO & Rh group as patient selected from blood bank
    • Avoid any other groups the patients has antibodies against (detected in screen- if patient has antibodies choose blood that’s negative for the antigens)
    • Crossmatching- patients plasma mixed with aliquots of donor RBCs- see if reac occurs (agglutination or haemolysis);
    o If no reac- RBC units compatible, no risk of acute haemolysis so can transfuse the unit
    o If reac- RBC units incompatible, risk of acute haemolysis so don’t transfuse the unit
How well did you know this?
1
Not at all
2
3
4
5
Perfectly
45
Q

Why we transfuse patients?

A
  • Prevent anaemia symptoms- TRUE
  • Improve quality of life of anaemic patients- TRUE
  • Prevent ischaemic damage of end organs in anaemic patients- TRUE
How well did you know this?
1
Not at all
2
3
4
5
Perfectly
46
Q

What is anaemia?

A

Anaemia- condition in which n.o. of RBCs/ their O2 carrying capacity insufficient to meet physiological needs (which vary be age, sex, altitude, smoking &preg status)
• Can’t measure amount of O2 carried, so measure Hb level to determine anaemia;
o Non preg women (15yrs &; above) <120g/L
o Men (15yrs & above) <130g/L

How well did you know this?
1
Not at all
2
3
4
5
Perfectly
47
Q

What is the Mechanism of Adaptation to Anaemia?

A
  • Increased cardiac output
  • Increased cardiac artery blood flow
  • Increased oxygen extraction
  • Increased RBC 2,3 DPG (disphosphoglycerate)
  • Increase EPO produc
  • Increase erythropoiesis
  • AIM of these mechanisms: maintain tissue oxygenation (prevent tissue hypoxia)- not all patients display symptoms, only those that can’t adapt to anaemia.
  • Acute anaemia- resp rate more markedly increases than in chronic anaemia.
  • Chronic anaemia- O2 extraction increased due to rise in 2,3 DPG, kidneys respond to hypoxia by increasing erythropoietin produc which results in increased erythropoiesis.
How well did you know this?
1
Not at all
2
3
4
5
Perfectly
48
Q

What are the symptoms of anaemia?

A

SYMPTOMS OF ANAEMIA= TISSUE HYPOXIA!!!
• So only patients that fail to use adaptation mechanisms to adjust to anaemia develop symptoms (not everyone)
• Not all patients develop symptoms of anaemia at same level of Hb- diff factors that affect adaptation mechanisms so each individual adapts diff (so don’t have 1 Hb threshold that determines if anaemic)

How well did you know this?
1
Not at all
2
3
4
5
Perfectly
49
Q

How we determine transfusion threshold that will prevent hypoxia in any individual patient?

A

Parameters That Affect Adaptation Mechanisms to Anaemia
• Underlying conditions that affect cardiac output, arterial blood flow, O2 saturation of Hb impair adaptation mechanisms to anaemia
• E.g. underlying conditions that can impair these mechanisms; cardiovasc diseases, resp diseases, age
• Patients with acute anaemia don’t have time to develop all adaptaion mechanisms so develop symptoms earlier than patients with chronic anaemia.

How well did you know this?
1
Not at all
2
3
4
5
Perfectly
50
Q

What do you need to consider when making the Clinical Decision to Transfuse?

A

Consider;
• Expected benefits; treat/ prevent anaemia & avoid long term effect of ischeamia symptoms
• Harms; adverse events of transfusion
• Cost; days of hospitalisation, use precious resource.
• Availability of alternatives (if can treat them, will treat them)
• Patient’s wishes (e.g. may refuse to be transfused)
• Clinical evidence
Then determine transfusion threshold &a target

How well did you know this?
1
Not at all
2
3
4
5
Perfectly
51
Q

How do you Determine Ideal Hb Conc for Various Groups/ Individual Patients?

A
  • Transfusion threshold (trigger)- Hb conc at or below which packed RBCs usually ordered for transfusion. Hb conc clinically decided based on holistic assessment of patient’s needs. (Lowest Hb conc not associated with anaemia symptoms)
  • Transfusions thresholds differ in various subgroups of patients depending on balance between mechanisms of adaptation to anaemia & O2 requirements.
  • Triggers of transfusion protect patients from unnecessary transfusions.
How well did you know this?
1
Not at all
2
3
4
5
Perfectly
52
Q

When is transfusion needed in Acute Anaemia ?

A

• Transfusion thresholds for patients with acute anaemia: Hb 70-80g/L EXCEPT symptomatic patients (transfused until symptoms relieved), patients with acute coronary syndrome, patients with major haemorrhage.
• Why we transfuse RBCs- to restore O2 carrying capacity
• Recommended not to transfuse patients with a treatable cause of anaemia e.g. iron deficiency.
• Triggers:
o ≤70 g/L for patients with mild symptoms of anaemia
o ≤80 g/L for patients with cardiovascular disease
• Alternatives to RBC transfusions:
o Correction of treatable causes of anaemia: iron deficiency, B12 & folate deficiency, erythropoietin treatment for patients with renal disease
o Correction of coagulopathy: discontinuation of antiplatelet agents, administration of anti-fibrinolytic agents

• If blood loss >40% of their total blood vol- need a transfusion

How well did you know this?
1
Not at all
2
3
4
5
Perfectly
53
Q

When is transfusion needed in Chronic Anaemia ?

A

• E.g. due to haematological syndromes.
• Consider setting individual thresholds & Hb conc targets for each patient who needs regular blood transfusions for chronic anaemia.
• Objectives for patients on regular transfusions due to myeloid failure syndromes;
o Relieve symptoms of anaemia
o Improvement of Quality of Life
o Prevention of ischemic organ damage
• Start with threshold Hb 80-100g/dl
• Tailor decision for each patient, consider;
o Co-morbidites that affect cardiac/ resp function,
o Risk for developing iron overload
o Adaptaion to anaemia (compensatory mechanisms developed).
• Patients on chronic transfusion programmes as patients with MDS that are NOT on chemo &; NOT thrombocytopenic usually transfused with target Hb 8-10g/dl.

• Objectives for patients on regular transfusions due to thalassaemia;
o Suppress endogenous erythropoiesis
• Threshold 90-95g/L, target 100-120g/L
• Consider iron overload!

How well did you know this?
1
Not at all
2
3
4
5
Perfectly
54
Q

Why Transfuse Patients with platelets?

A
Platelets 
•	Stored at room temp (22°C)
•	Shelf life 5 days from collection
•	‘Adult therapeutic dose’- platelets from 4 pooled donations (or equivalent n.o. from a single apheresis donation)
•	Usual transfusion time: 30min/unit. 

Why Transfuse Patients?
• Treatment of bleeding due to severe thrombocytopenia (low platelets) or platelet dysfunction
• To prevent bleeding
• We transfuse;
o Massive haemorrhage (keep platelet count above 75 x 109/L)
o Bone marrow failure (platelet count <10 x 109/L or <20 x 109/L, e.g. sepsis)
o Prophylaxis for surgery (more major surgery 50 x 109/L, CNS or eye surgery 100 x 109/L)
• Contraindications;
o Heparin induced thrombocytopenia &thrombosis
o Thrombotic thrombocytopenic purpura

How well did you know this?
1
Not at all
2
3
4
5
Perfectly
55
Q

What is Fresh Frozen Plasma? What is it used for? What is it contraindicated in?

A

Fresh Frozen Plasma (1 unit of FFP)
• Stored at -30°C for up to 24 mnths
• Thawed immediately before use (takes 20-30 min)
• Usual dose 12-15mL/kg (4-6 units for average adult)
• Usual transfusion time: 30mins/unit
• Main indications;
o Coagulopathy with bleeding/ surgery
o Massive haemorrhage
o Thrombotic thrombocytopenic purpura
• Do NOT transfuse fresh frozen plasma with;
o Warfarin reversal
o Replacement of single factor deficiency e.g. haemophilia

How well did you know this?
1
Not at all
2
3
4
5
Perfectly
56
Q

What is Prothrombin Complex Concentrate?

A

• Product used to reverse warfarin is Prothrombin Complex Concentrate (PCC);
o Plasma- derived vit K dependant factors (II, VII, IX, X)
o For emergency reversal of life-threatening warfarin over-anticoagulation (do not use FFP)
o Issued by transfusion lab- supply in A&;E

How well did you know this?
1
Not at all
2
3
4
5
Perfectly
57
Q

Why are transfusions kept CMV negative?

A
CMV NEGATIVE
•	To keep at-risk patients CMV free (50% of us CMV –ve)
•	Children <1 yr
•	Intrauterine transfusions 
•	Congenital immunodeficiency 
•	And unless known to be CMV IgG +ve
•	Preg women having elective transfusion
How well did you know this?
1
Not at all
2
3
4
5
Perfectly
58
Q

Why are transfusions irradiated?

A

IRRADIATED
• To prevent transfusion-associated graft vs host disease (rare) in specific T-cell immunodeficiency cases
• Intrauterine transfusions
• Congenital immunodeficiency
• Hodgkin lymphoma
• Stem cell/ marrow transplant patients
• After purine analogue chemo (e.g. fludarabine)

How well did you know this?
1
Not at all
2
3
4
5
Perfectly
59
Q

What are Acute Transfusion Reactions ?

A

Acute Transfusion Reactions (present <24hrs of transfusion)
Immunological;
• Acute haemolytic transfusion reac due to ABO incompatibility
• Allergic/ anaphylactic reac
• Transfusion related acute lung injury (TRALI)
Non immunological;
• Bacterial contamination
• Transfusion associated circulatory overload (TACO)
• Febrile non-haemolytic transfusion reac

How well did you know this?
1
Not at all
2
3
4
5
Perfectly
60
Q

What are Delayed Transfusion Reactions ?

A

Delayed Transfusion Reactions (present >24hrs of transfusion)
Immunological;
• Transfusion-associated graft vs host disease (TA-G v HD)
• Post transfusion purpura
Non immunological
• Transfusion Transmitted Infection (TTI)- viral/prion
• Iron overload

How well did you know this?
1
Not at all
2
3
4
5
Perfectly
61
Q

What are the Infective Risks of Blood Transfusion?

A

Infective Risks of Blood Transfusion
• Risk of transmitting Hep B/C/HIV viruses are low
Prion disease
• E.g. Crutzfeld Jacob disease
• Steps to ↓ risk of potential transmission by blood/ blood components; leucodepletion, UK plasma not used for fractionation, imported FFP for patients born after 1996.

How well did you know this?
1
Not at all
2
3
4
5
Perfectly
62
Q

What is ABO Incompatibility?

A

Acute Haemolytic Reac-ABO Incompatibility
• Due to transfusion of RBCs that express ABO antigens to a recipient that has preformed antibodies against these antigens.
• Antibodies will bind on RBC surface &; be haemolysed intravascularly.
• Due to release of free Hb (toxic) in circ;
o Hb deposited in distal renal tubule= acute renal failure
o Coagulation stim = microvascular thrombosis
o Stim cytokine storm
o Scavenges NO = generalized vasoconstriction

  • Severe reacs may occur within 15min of transfusion
  • Milder reacs may occur later but usually before end of transfusion
  • 20-30% fatal
Acute Haemolytic Reac- ABO Incompatibility SIGNS &amp; SYMPTOMS
•	Fever &amp;; chills
•	Back pain
•	Infusion pain
•	Hypotension/ shock
•	Hemoglobinuria (may be 1st sign in anaethsized patients)
•	Increased bleeding (DIC)
•	Chest pain
•	Sense of ‘impending death’
How well did you know this?
1
Not at all
2
3
4
5
Perfectly
63
Q

What are Delayed Haemolytic Reactions?

A
  • Due to immune IgG antibodies against RBC antigens other than ABO
  • The antibodies formed after transfusion
  • Other groups on the surface of RBCs such as Rh, Duffy (Fy), Kell(Kk), Kidd (Jk) and La
Delayed Haemolytic Reac- CLINICAL FEATURES 
•	Onset 3-14 days after RBC transfusion
•	Fatigue
•	Jaundice
•	And/or fever 

Delayed Haemolytic Reac- Lab Findings
• Drop in Hb
• Increased LDH (sign of haemolysis)
• Increased indirect bilirubin (sign of haemolysis)
• DIRECT ANTIGLOBULIN TEST (Coomb’s Test) +VE

How well did you know this?
1
Not at all
2
3
4
5
Perfectly
64
Q

What is Coomb’s test?

A

The Anti-human Globulin (Coomb’s) Test
• (Test has diff names; Coomb’s test, Anti-human globulin test (AHG), Direct Anti-globulin test (DAG)).
• To detect incomplete (IgG) antibodies
• In 1945 Coombs raised rabbit antibodies to human IgG (anti-human globulin or AHG)
• These used to detect IgG antibodies on RBCs
• Normal RBCs don’t have anything on surface, but if covered with antibodies reagent will create bridges between RBCs & they will agglutinate.
• Key test in blood transfusion

How well did you know this?
1
Not at all
2
3
4
5
Perfectly
65
Q

What is TRALI?

A

Transfusion Related Acute Lung Injury (TRALI)
• Antibodies in transfusion
• Estimated rate of fatalities 5-10%
• Donor has antibodies to antigens on recipient’s leucocytes- bind &; activate them causing them to release intracellular substance = activate endothelium.
• Almost always complicates transfusion of plasma rich components (platelets, FFP)

How well did you know this?
1
Not at all
2
3
4
5
Perfectly
66
Q

What is the Criteria for TRALI Diagnosis ?

A

• Sudden onset of ‘Acute Lung Injury’ within 6 hrs of transfusion
• Acute lung injury;
o Hypoxemia
o New bilateral chest x-ray infiltrates (due to fluid capill leak)
o No evidence of vol overload (heart size normal

How well did you know this?
1
Not at all
2
3
4
5
Perfectly
67
Q

What is the treatment for TRALI?

A

TRALI- Treatment, Clinical Course
• Majority of patients recover within 72-96hrs
• Mild forms of TRALI may respond to supplemental oxygen therapy.
• Severe forms may require mechanical ventilation and ICU support.
• Lab investigations; donor tested for HLA &granulocyte antibodies, recipient tested for neutrophil antigens
• Diagnosis confirmation; donor has antibodies against antigens that are expressed on recipients granulocytes

How well did you know this?
1
Not at all
2
3
4
5
Perfectly
68
Q

What is Transfusion-Associated Circulatory Overload (TACO)?

A
TACO Signs &amp;; Symptoms
•	Symptoms; 
o	Sudden dyspnea
o	Orthopnoea
o	Tachycardia
o	Hypertension
o	Hypoxemia 
•	Signs;
o	Raised BP
o	Elevated jugular venous pressure 
TACO Risk Factors
•	Elderly patients
•	Small children
•	Patients with compromised L ventricular function
•	Increased vol of transfusion
•	Increased rate of transfusion 

TACO significantly underreported & often unrecognised by clinicians.

How well did you know this?
1
Not at all
2
3
4
5
Perfectly
69
Q

What are allergic reactions in response to transfusions?

A

• Due to antibodies in recipient’s serum against transfused proteins
• Usually after transfusion of plasma or platelets
• URTICARIAL RASH +/- wheeze;
o Often not severe
o Hypersensitivity to a ‘random’ plasma protein
• ANAPHYLAXIS
o Usually in patients that are IgA deficient & have anti-IgA antibodies
o Severe life-threatening reac soon after transfusion started
o Wheeze/asthma, high pulse, low BP (shock)
o With laryngeal oedema/ facial oedema
• Lab investigations: IgA quantification, anti-IgA antibodies testing

How well did you know this?
1
Not at all
2
3
4
5
Perfectly
70
Q

What are Febrile Non-haemolytic Transfusion Reactions (FNHTR)?

A

• Pyrexial reac
• During or soon after transfusion;
o Fever- rise in temp >1°C +/- shakes/ rigors
o +/- high pulse
• Unpleasant but not life threatening
• FNHTR due to cytokines/ other biologically active molecules that accumulate during storage of blood components.
• Less since leucodepletion of blood & platelets
• Discontinue transfusion until exclude ‘wrong blood’ or bacterial infec
• Self-limited reac (stop when stop transfusion)

How well did you know this?
1
Not at all
2
3
4
5
Perfectly
71
Q

What is Haemostatic Plug Formation?

A

Haemostatic Plug Formation
• Primary haemostasis- platelet plug formation &; vascular constric
o 1st line of defence to stop bleeding
o Reac of vessel wall
o When vascular integrity of blood vessels breached= blood exposed to collagen (underlying vessel)= platelets are activated=form a plug at site of injury in vessel
• 2ndry event (in fluid phase of blood) - coagulation cascade (makes clot)
o Series of sequential enzymatic reacs form active proteins to enzymes form fibrin clot which stabilises platelet clot
• Both primary &; 2ndry processed work together to form a haemostatic plug- prevent further bleeding

How well did you know this?
1
Not at all
2
3
4
5
Perfectly
72
Q

What is Platelet Adhesion & Activation ?

A

Platelet Adhesion & Activation
• When normal platelets in blood- inert
• When exposed to collagen in vessel wall after injury, von willie brand factor (blood protein) adheres to surface of collagen &; also binds to recep on platelet- forms glue between platelet & vessel wall
• This interaction 1st step that happens in platelet activation
• Platelets activated in result of adherence to vessel wall=produce new receps on surface e.g. glycoprotein 2b3a recep undergoes conformational change into a active fibrinogen recep
• This allows fibrinogen to bind to platelets (fibrinogen in blood forms bridge between platelets- keeps platelets together)
• Alpha granules & dense granules in platelets- releases contents (adhesion proteins)when platelets activated= +ve feedback loop to activate other platelets
• Thromboxane synthesised inside platelets; is a procoagulant proplatelet activating material, activates further platelets
• Builds up so more platelets activated= plug formation
• Receps for other small molecules (e.g. ADP)- bind to other activators in circulation

How well did you know this?
1
Not at all
2
3
4
5
Perfectly
73
Q

What is Thromboxane synthesis?

A

Thromboxane synthesis
• Cyclooxygenase enzyme leads to thromboxane synthesis
• Aspirin (anti-thrombotic) inhibits cyclooxygenase inside platelets reducing thromboxane synthesis
• Clopidogrel (antiplatelet)- antagonist at platelet ADP recep
• Also large protein 23ba antagonists (fibrinogen recep)- stop platelets aggregating (in patients with acute coronary syndromes)

How well did you know this?
1
Not at all
2
3
4
5
Perfectly
74
Q

What is the clotting cascade? Intrinsic and extrinsic?

A

Clotting Cascade
• 3 parts to clotting cascade; extrinsic, intrinsic & common pathway (tests look at each pathway)
• Prothrombin time- looks at extrinsic pathway
• APTT- looks at intrinsic pathway
• Thrombin clotting time- looks at common pathway

Extrinsic Pathway (& common pathway)
• Most important initiator of coag cascade
• Extrinsic & common system (most common way fibrin made in vivo)
• Not many steps- fast way of making thrombin & fibrin
1. Initiated when vessel damaged- cells in vessel wall express tissue factor on surface of cell (so not circulating in blood)
2. Tissue factor activates factor 7 (7a)
3. 7a + tissue factor activate factor 10 (to factor 10a)
4. F10a + F5 (cofactor) on phospholipid surface (activated platelet) activates F2 (prothrombin) to thrombin
5. Thrombin cleaves fibrinogen= fibrin monomers= polymerise to form fibrin clot

Intrinsic Pathway
• Intrinsic (doesn’t need external protein from fluid in blood unlike extrinsic where need tissue factor).
• Intrinsic pathway has more steps than extrinsic- slower way of getting fibrin
1. F12 activated when blood come into contact with foreign surface
2. F12a activates F11 to F11a,
3. F11a activates F9 to 9a (need phospholipid surface)
4. F9a with cofactor F8 activates factor 10 (phospholipid surface provided by activated platelets)
5. (Then back to common pathway- thrombin & fibrin formation)

  • Deficiency of factors 8 & 9 in intrinsic pathway= haemophilias
  • Patients deficient in F12 no bleeding disorder
  • Deficient in f11- variable bleeding; so top end of intrinsic pathway may not be as important as extrinsic (but deficiencies of F8 & 9 is important- haemophilais)
  • We think in vivo, extrinsic pathway quickly activated when vasc damage, small amounts of thrombin generated which feedsback into intrinsic pathway & directly converts F11 into 11a (without need for 12b) & also activates F8.
  • On phospholipid surface 7a can activate F9- so these intrinsic & extrinsic pathways not completely separate BUT lab tests (in unphysiological env) can sep into intrinsic, extrinsic & common pathways
  • Cross between platelet activation & 2ndry part of haemostasis
How well did you know this?
1
Not at all
2
3
4
5
Perfectly
75
Q

What are Anticoagulant mechanisms ?

A
  • Natural anticoagulant protions; protein C & S and anti-thrombin III, & fibronolytic system (when clot forms body will slowly dissolve it)
  • Fibrinolytic system- series of steps inert proteins activated
  • End product of finbronlytic cascade = plasmin (breaks down fibrin clots)
  • Plasmin from plasminogen & TPA.
  • Alpha 2 anti-plasmin & Pi1 (inhibitor of plasminogen activator TPA), present in plasma- stop fibrinolytic system being activated in fluid phase of blood &localise fibrinolysis to surface of clots (ensures plasmin doesn’t start digesting & dissolving coagulation factors).
How well did you know this?
1
Not at all
2
3
4
5
Perfectly
76
Q

What are the different types of Bleeding Disorders ?

A
  • Congenital: usually single defect- 1 of factors/ platelet proteins will be deficient, or von willie brand factor deficient
  • Acquired: often multiple defects, several parts of haemostatic system affected e.g. lots of factors
  • Platelet/ bleeding vessel wall: mucosal & skin- skin bleeding, bruising, mucosal bleeding e.g. epistaxis
  • Coag defect (deficiency in 1 or more of coag proteins or enzymes): deep muscular &; joint bleeds, bleeding following trauma, deep tissue bleeding into joints & muscles e.g. in haemophilias
How well did you know this?
1
Not at all
2
3
4
5
Perfectly
77
Q

What causes defects in primary haemostasis?

A

Platelet/ Vessel Wall Defects
• All give rise to prolonged bleeding time
• Reduced platelet n.o. (not got enough platelets)
• Abnormal platelet function (drugs/ aspirin- antiplateltes)- platelets not working
• Abnormalities of vessel wall e.g. inherited- ehlers dan los syndrome (cause weakness in vessel wall), acquired- vit C deficiency (need vit c to make good collagen- scurvy is an acquired bleeding disorder)
• Abnormal interaction between platelets &vessel wall (von willie brand disease- lack of protein that causes adhesion to vessel wall)

How well did you know this?
1
Not at all
2
3
4
5
Perfectly
78
Q

What is Petechiae ?

A
  • Do not blanch with pressure (e.g. angiomas)

* Not palpable (e.g. vasculitis)

How well did you know this?
1
Not at all
2
3
4
5
Perfectly
79
Q

What is Epistaxis ?

A

Epistaxis
• Mucosal bleeding
• If single symptom with nothing else- due to local cause in nose
• Often part of symptoms patients with primary haemostasis (e.g. von wilie brand disease) will have

How well did you know this?
1
Not at all
2
3
4
5
Perfectly
80
Q

What is disease Von Willie brand disease?

A

Von Willie brand disease- (defective PRIMARY haemostasis)
• Most common heritable bleeding disorder
• Men &; women affected
• Mucocutaneous bleeding main symptom including menorrhagia
• Post operative &post partum bleeding
• Variable penetrance for mild types
• Diagnosis of mild vWD difficult due to confounding factors e.g. exercise, stress &; illness
• Deficiency of von willie brand protein (bridge between platelets &collagen in vessel wall)
• Parts of it’s struc binds to a glycoprotein on platelets surface
• Consists of lots of subunits- large multimeric struc (lower molecular weight multimers not as good of a glue as high molecular weight- act as a better glue)
• Is a heterogeneous disorder- has diff types;
o TYPE 1 common type is a mild reduc in amount of VWBF produced (reduced amount of the normal protein), may have slightly reduced level of factor 8
o VWBF levels go up and down during the day in everyone e.g. go up if you have infec or after the gym, stress etc- so choosing right time to test people for the disease important &; need to retake test (1 result not representative)
o TYPE 2 normal amounts of ABNORMAL von wilie brand protein (doesn’t have enough high molecular weight forms- dispoportinatley reduced), in lab see disproportion between VWB factor antigen levels & activity levels (how well its working)
o Type 1 &; 2 VWB disease- autosomal DOMINANT inheritance
o Type 1 has variable penetrance in families
o Blood group O- have slightly lower levels of VWBF (doesn’t mean they have the disease)
o TYPE 3/ SEVERE VWB disease- ABSENT VWBF, more severe; e.g. spontaneous bleeding events
o VWBF also carrier for Factor 8 in plasma, so type 3 VWB disease will also have low F8 level too (not because they can’t make F8 but because molecule carrying F8 not there)- so may have coag disorder clinical manifestations
o VWB disease most common reason why a female will have low factor 8 level.

• 1 in 2 chance of passing it onto child regardless of sex of child but variable penetrance (some people may not get manifestation if have mild form of the disease)

Treatment
• Most patients don’t need treatment in day to day life, need treatment when got symptoms/ being challenged e.g. at dentist, surgical operation, having a baby, heavy period
• Antifibrinolytics: tranexamic acid- (e.g. for simple dental work, heavy periods, stop a nose bleed)
• Desmopressin (DDAVP) given by subcutaneous inject (vasopressin hormone; sets tone of smooth muscle in blood vessels &; water balance (helps us to hang onto water)- helps to release VWBF stores into circ, temporarily raising VWBF in blood)- type 1 of this disease respond to this treatment
• Desmopressin temporary, if repeat it too many times then patient will stop responding to it (tachyphylaxis) (never give it more than 3 times), it also causes water retention so need to watch amount of water taking &; fluid restrict patient
• Factor concentrates containing vWD (plasma derived blood donations where plasma sep by being centrifuged)
• To get vaccinated against hep as with treatment would be exposed to pooled plasma (plasma form diff blood donations)- when treatment that exposes them to diff types of blood.
• COCP (combined oral contraceptive pill) for menorrhagia- causes women to have lighter periods

How well did you know this?
1
Not at all
2
3
4
5
Perfectly
81
Q

How do results of clotting tests help determine which factors are affected?

A
  • These clotting screening tests not diagnostic- tell us something wrong but not what’s wrong
  • But they can narrow down what specific factors want to assay.
  • E.g. if get isolated prolongation of APTT want to isolate factors in intrinsic pathway (F 9, 8, 11 & 12)
  • If isolated elongation of prothrombin time want to isolate F7
  • If both abnormal- assay all of them (may be single defect in common pathway or could be more than 1 factor deficiency.
  • F13 deficiency &; VWB disease (if don’t have F8 deficiency)- expect normal tests (deficiency screens won’t pick these up e.g. F13 comes after clot formed)
How well did you know this?
1
Not at all
2
3
4
5
Perfectly
82
Q

What are Heritable Coagulation Factor Deficiencies?

A

• F14- stabilises fibrin clot (comes after clot formation), covalently links fibrin monomers (stabilisin them)
• Deficiencies from factors all from single gene defects
• Most common disorders haemophilia A (F8 deficiency) &haemophilia B (F9 deficiency);
o Are x-linked
o Level of deficiency of these factors in blood correlate with clinical manifestations
F11 deficiency autosomal dominant or recessive

XII- Autosomal common
XI- Autosomal rare
VII- Auto recessive V.R
X,V,II,I, XIII- Auto recessive V.R

How well did you know this?
1
Not at all
2
3
4
5
Perfectly
83
Q

What is Haemophilia A & B ?

A
Haemophilia A &amp; B 
•	F8 &amp;9 intrinsic pathway
•	F9- serene protease enzyme
•	F8- cofactor 
•	APTT looks at intrinsic pathway so do APTT lab test- isolated prolongation of APTT in clotting screen picks up haemophilia  

• Haemophilia A &; B CLINICALLY INDISTINGUISHABLE
• X linked recessive inheritance; females carriers- have about 50% of normal F8 would expect which is enough F8 to get by, don’t have bleeding disorder, some have less than 50% so have mild haemophilia (as 1 X chromosome normal & other carrying defective gene- non-random lyonisation is why some people have lower levels of F8 as normal X predominantly inactivated)
• Typically females carriers & males affected
• Haemophila A;
o Factor VIII deficiency
o 1 in 5,000 males
• Hemophilia B;
o Factor IX deficiency
o 1 in 30,000 males (more rare)
• Severity consistent between family members- if fam has history of severe haemophilia child will get severe, if mild history then child will get mild haemophilia.
• 30% haemophilia A are new mutations (no fam history)

How well did you know this?
1
Not at all
2
3
4
5
Perfectly
84
Q

What are the Degrees of Severity of Haemophilia ?

A
  • Normal F 8 or 9 level= 50-150%
  • Mild haemophilia F8 or 9 level= 6-50% (won’t spontaneously bleed, only bleed after trauma/ surgery)
  • Moderate haemophilia F8 or 9 level= 1-5% (bleed following minor trauma, 1 or 2 spontaneous bleeds)
  • Severe haemophilia F8 or 9 levels= <1%, common bleeds (muscle &; joint bleeds) e.g. often presents early in children e.g. when learning to walk
  • General rule: less severe haemophilia gets- later in life will present &; be diagnosed haemophilia
How well did you know this?
1
Not at all
2
3
4
5
Perfectly
85
Q

What are the Types of Bleeds in Haemophilia?

A
  • Spontaneous/ Post traumatic
  • Joint bleeding (swollen, hot)= haemarthrosis
  • Muscle haemorrhage- muscle fibres become damaged as a result of bleed
  • Soft tissue (floor of mouth/ palate) e.g. haematomas under skin, retroperitoneal bleeding
  • Life threatening bleeding (e.g. intercranial bleeding, bleeding around airway- can obstruct airway)
How well did you know this?
1
Not at all
2
3
4
5
Perfectly
86
Q

What is the treatment for Haemophilia?

A
  • Replace deficient factor (missing clotting protein) using recombinant factor (not blood derived)- on demand or prophylaxis (prevention of bleeds)
  • DDAVP (mild/ moderate Haemophilia A)
  • Factor concentrates: Recombinant are products of choics
  • Antifibrinolytic agents
  • Vaccination against hep A &; B
  • Supportive measures e.g. icing (ice pack), immobilisation, rest

Heamophilia A &; B Management
• Specialised centres; haemophilia centres &; comprehensive care centres, multidisciplinary approach, home treatment, patient education & social support, physio, orthopaedic advice &; treatment, treatment & diagnosis of liver disease, specialised management for HIV +ve patients, genetic counselling &parental diagnosis.
Ask haemophila patients who their haemophila centre is & talk to their centre.

How well did you know this?
1
Not at all
2
3
4
5
Perfectly
87
Q

What are the complications of treatment for Haemophilia?

A

Complications of Treatment of Haemophilia
• Transfusion transmitted infec; hep A, B, C etc, HIV, Parvovirus, vCJD
• Inhibitor development;
o Inhibitor- alloantibody against factor 8 or 9
o 25% in haemophilia A (more common in haemophilia A than B)
o Result in poor recovery and/or shortened ½ life of factor replacement therapy
o Poor clinical response to treatment- specialised management of bleeds
o Most occur early in course of condition (10 exposure days)
o Eradication of inhibitor (immune tolerance)
o Genetic predisposition
o Some transient (go away on their own), but some not transient- render treatment ineffective
o IF get inhibitor- more likely to bleed.

How well did you know this?
1
Not at all
2
3
4
5
Perfectly
88
Q

What suggests a vascular/ platelet defect?

A

Petechiae and superficial bruises
skin and mucous membranes
spontaneous
bleeding immediate prolonged and non-recurrent

How well did you know this?
1
Not at all
2
3
4
5
Perfectly
89
Q

What suggests a coagulation defect?

A

Deep spreading haematoma
Haemarthritis
Retroperitoneal bleeding
Bleeding prolonged and often recurrent

How well did you know this?
1
Not at all
2
3
4
5
Perfectly
90
Q

What are Acquired Bleeding Disorders ?

A
  • Vit K deficiency
  • Liver disease
  • Massive transfusion syndrome
  • Disseminated Intravascular Coagulation
  • Iatrogenic
  • Acquired inhibitors (antibodies that affect particular clotting factor/ mechanism)
How well did you know this?
1
Not at all
2
3
4
5
Perfectly
91
Q

How do you Assess Patients with Possible Bleeding Disorders?

A
  • Clinical history important
  • Date of onset, previous history of bleeding episodes &; clinical pattern
  • Response to challenges e.g. surgery, dental extraction
  • For young children; bleeding from umbilical stump, vaccinations, circumcision
  • Requirement for medical/ surgical intervention
  • Other systemic illness &; drug history
  • Fam history
  • Clinical examination: pattern of any bruising/ other evident haemorrhangic signs, digns of underlying disease, joints, muscles & skin.
How well did you know this?
1
Not at all
2
3
4
5
Perfectly
92
Q

How do you test for Possible Bleeding Disorders?

A

Tests
• Collect blood into tube that contains sodium citrate (removes sodium calcionate)- inhibits sample clotting in test tube
• Smaple in centrifuge in lab- all cellular components of blood drop to bottom, left with straw-coloured plasma at top.
• Do coag screening on plasma (with no platelets or RBCs in)
• Then do prothrombin time-……..
• Time how long it takes for prothrombin to form
• Recalcify & add substitiue factor(?)

• Prothrombin time
• APTT test;
o Add particular material
o Don’t add calcium
o Measure for certain length of time for clot to form- activation of factor 12 &; 11a.
• Thrombin clotting time (final step in common pathway);
o Add activated factor 2 (thrombin)- expect it to cleave fibrinogen….
o Don’t need to calcify
• Fibrinogen level
• Mixing test…

How well did you know this?
1
Not at all
2
3
4
5
Perfectly
93
Q

How to test whether its a Deficiency or Inhibitor?

A
  • APTT prolonged
  • Repeat with 50:50 mix patient to normal plasma
  • Significant correction: deficiency
  • No correction: inhibitor
How well did you know this?
1
Not at all
2
3
4
5
Perfectly
94
Q

What are common blockers of the Coagulation Cascade ?

A

• Gamma glutamyl carboxylase enzyme
• Warfarin- anticoagulant (can’t recycle vit k so eventually become vit K deficient)
• PIVKA- proteins induced by vit K absence
• Efficacy/ safety ratio for currently available therapies less than satisfactory due to their ill-defined, multitargeted activity
• Current available antithrombotic agents; heparins (UFH &Enoxaparin), vit K antagonists (warfarin) &; direct thrombin inhibitors (hirundins)
• Most used- heparin & vit k; have a range of actions on diff parts of the coag cascade (unpredictable clinical responses)
• Other limitations of antithrombotics-
o High incidence of serious adverse effects e.g. bleeding complications
o Monitor coag markers- burden of time & cost
o Narrow therapeutic margin
o Limited effectiveness in preventing VTE
• Factor Xa inhibitors- antithrombic agents designed to selectively target 1 core step in coag cascade leading to potent &; targeted effectiveness.

How well did you know this?
1
Not at all
2
3
4
5
Perfectly
95
Q

What happens as a result of Vit K deficiency ?

A

• Deficiencies of factors II, VII, IX, X
• Are fat soluble vits (stored in body longer than water soluble vits)
• Causes;
o Obstructive jaundice
o Prolonged nutritional deficiency
o Broad spectrum antibiotics
o Neonates (classical 1-7 days)- in UK given vit K to avoid haemorraghic disease of the newborn

How well did you know this?
1
Not at all
2
3
4
5
Perfectly
96
Q

What happens as a result of Liver Disease ?

A
  • Cirrhotic coagulopathy major source of morbidity & mortality in patients with liver disease
  • Increased risk of severe bleeding form invasive procedures/ surgery
  • Conventional methods for treatment &prevention of bleeding are limited (may not increase deficient factors to hemostatic range, required vol for transfusion may be too large, theoretical risk of viral transmission)
  • Thrombocytopenia, prolonged PT & hyperfibrinolysis- most freq haematological abnormalities in this pop
  • Cirrhotic patients with prolonged PT have low levels of vit-k dependant coag factors- role in bleeding tendency
  • Platelet dysfunction (plasma induced cleavage of surface glycoproteins)
  • Reduced plasma conc of all coag factors (reduced synthesis) except FVIII
  • Delayed fibrin monomer polymerisation due to altered fibrinogen glycosylisation (xs sialic acid)
  • Excessive plasmin activity
How well did you know this?
1
Not at all
2
3
4
5
Perfectly
97
Q

What is a Massive Transfusion ? What Haemostatic Abnormalities are present?

A

Massive Transfusion
• Transfusion of vol equal to patients total blood vol in less than 24hrs or
• 50% blood vol lost within 3 hours
Haemostatic Abnormalities in Massive Transfusion
• Due to dilutional depletion of platelets &coagulation factors
• Due to DIC (due to tissue damage)- risk factors extensive trauma, head injury, prolonged hypotension
• Due to underlying disease e.g. liver or renal drug treatment or surgery

How well did you know this?
1
Not at all
2
3
4
5
Perfectly
98
Q

What are the ?Dilutional Effects on Haemostasis

A
  • Citrate used as anticoagulant in RBCs
  • Thrombocytopenia- at least 7-8 litres in adults usually transferred before probs likely
  • Coag factor depletion- mainly factors V & VIII & fibrinogen
  • DIC common
  • Citrate toxicity uncommon- hypothermia & neonates- increases susceptibility
  • Hypocalcaemia =- no clinically signif effect on coag
How well did you know this?
1
Not at all
2
3
4
5
Perfectly
99
Q

What is the Pathophysiology of DIC ?

A
  • Occurs because of a trigger to activate coagulation cascade in vivo.
  • Acute DIC
  • Chronic DIC
  • Activation of fibrinolytic system
  • Generalised disruption in physiological balance of procoag &; anticoag mechanisms
  • Consumption of clotting factors &; platelets
  • Microvascular thrombosis (tissue ischaemia & organ damage)
  • Fibrinolysis activation
  • Microangiopathic haemolysis
How well did you know this?
1
Not at all
2
3
4
5
Perfectly
100
Q

What are the causes of Acute DIC?

A
  • Sepsis
  • Obstetric complications
  • Trauma/ tissue necrosis
  • Acute intravascular haemolysis e.g. ABO incompatible blood transfusion
  • Fulminant liver disease
How well did you know this?
1
Not at all
2
3
4
5
Perfectly
101
Q

What are the causes of Chronic DIC?

A
  • Malignancy
  • End stage liver Disease
  • Severe localised intravascular coagulation
  • Obstetric- retained dead foetus
How well did you know this?
1
Not at all
2
3
4
5
Perfectly
102
Q

What are the lab tests for DIC?

A
  • Rarely get DIC without thrombocytopenia
  • Fibrinogen conc usually lower (may not be pathologically lowered)
  • FBC &; blood film
  • Coag screen; PT (70% prolonged), APTT (50% prolonged), TCT (usually prolonged), fibrinogen conc, FDP or D-dimer (elevated in 85%).
  • Not 1 single diagnostic test- scoring systems sometimes used
How well did you know this?
1
Not at all
2
3
4
5
Perfectly
103
Q

What is the management of DIC?

A
Management of DIC
•	TREAT UNDERLYING CAUSE;
o	Antibiotics
o	Obstetric intervention
o	Chemo/ ATRA/ tumour resection 
•	SUPPORTIVE TREATMENT;
o	e.g. give fresh frozen plasma 
o	Maintain tissue perfusion
o	Co-ordinate invasive procedures
o	(Folic acid &amp; vit K to support recovery period especially if illness prolonged) 

Product Support in DIC
• Determined by bleeding/ inavasaive procedure
• Platelet tranasfusion if platelets <50
• PT or APTT ratio >1.5:FFP 15ml/kg
• Fibrinogen <1g/l: Cryoprecipitate or Fibrinogen concentrate

104
Q

What Anticoagulant induced bleeding ?

A
  • Warfarin
  • If want to reverse effects of warfarin, want to give vit K- sometimes need to do more if has life threatening bleeding (vit k takes several hrs to work), can give a 4 factor concentrate (PCC)- immediate edge to reverse warfarin
  • Heparin- combines antithrombin & enhances its effects, antithrombin inhibit F2a & F10a
  • Neuro anticoagulants being commonly used; agents that are orally active, work in a target specific way n coag cascade e.g. riberoxiban & epixiban (direct 10 a inhibitors), also F2a inhibitors- can use to treat DVTs
105
Q

What are VKA Oral Anticoagulants?

A
  • Control of dosing by INR
  • INR= prothrombin ratio
  • Prothrombin ratio= ratio of patients prothrombin time/ mean normal prothrombin time
  • ISI= correction factor to account for sensitivity of thromboplastin compared with international reference preparation (IRP)
106
Q

Which drugs can potentiate warfarin effect?

A
Drugs which can potentiate warfarin effect;
•	Cimetidine			
•	Ampicillin (oral)
•	Amiodarone		
•	NSAID’s
•	Sulphinpyrazone	
•	Chlorpromazine
•	Cotrimoxazole		
•	Sulphonylureas
•	Erythromycin		
•	Corticosteroids
•	Cephlosporins
107
Q

Which drugs can antagonise warfarin effect?

A
Drugs which antagonise warfarin effect;
•	Cholestyramine
•	Spironolactone
•	Rifampicin
•	Carbamazepine
•	Vitamin K
108
Q

What are the Recommendations for Reversal of VKA Oral Anticoagulant Treatment?

A

• Life threatening haemorrhage
o 5mg-10mg vit K IV
o 4 factor concentrate (PCC) e.g. Octaplex or Beriplex
• Non major bleeding
o Withhold warfarin, give vit K 1-3mg IV
• INR of >0.8 without haemorrhage
o Withold warfarin, give vit K 1-5mg orally
• INR >0.5 <0.8, without haemorrhage
o Withold warfarin, consider oral vit K if high risk for bleeding
• Unexpected bleeding at therapeutic levels
o Reverse warfarin appropriately, investigate underlying disease e.g. unsuspected renal/ alimentary disease

109
Q

What is Heparin Monitoring ?

A

• Anticoag effects of UFH can be monitored by tests sensitive to anti-thrombin & /or anti-Xa effects of heparin
• For purposes of full anticoag with UFH, the APTT most commonly used assay
• Alternatives:
o Heparin level by protamine titration
o Heparin level by anti-Xa assay
o Calcium thrombin time
o In situations where want to monitor LMWH effect- anti-Xa assays used
o APTT not sensitive to LMWH

• HEPARIN
o Stop infusion
o Consider protamine administration 1mg neutralises 100IU of heparin (max 40mg)
o Effects of protamine on LMWH- less predictable & complex

110
Q

What are the benefits of LMWH v UFH?

A

Properties of LMWH v UFH
• LMWH: higher ratio of anti-Xa to anti-IIa activity
• Improved bioavailability
• Longer ½ life allowing once daily administration
• More predictable anticoagulant response: monitoring not routinely recognised

111
Q

What are Chronic Myeloproliferative Disorders ?

A

• Clonal stem cell disorders of the bone marrow
• Malignant
• Conditions they cause in patient;
o Polycythaemia Vera (PV);
-Increased RBCs, +/- neutrophis, +/- platelets
- Distinguish from 2ndry polycythemias & relative polycythaemia
o Essential thrombocytosis;
- Increased platelets
- Distinguish from reactive thrombocytosis
o Idiopathic Myelofibrosis
- Variable cytopenias with large spleen
- Distinguish from other causes of splenomegaly

112
Q

What is Polycythaemia Vera?

A

Chronic Myeloproliferative Disorders
Increased RBCs, +/- neutrophis, +/- platelets
- Distinguish from 2ndry polycythemias & relative polycythaemia
•All ages- peak at 50-70yrs

113
Q

What are the signs and symptoms of Polycythaemia Vera?

A
•	Symptoms;
o	Insidious 
o	Itching (aquagenic- hot baths) 
o	Plethoric face (red face)
o	Headache, muzziness
o	General malaise 
o	Tinnitus 
o	Peptic ulcer
o	Increased call turnover= gout
o	Gangrene of the toes 
•	Signs;
o	Plethora 
o	Engorged retinal veins 
o	Splenomegaly  
•	Engorged retinal veins &amp; haemorrhages 
•	Can get polycythemia due to altitude (live in high places)
114
Q

How do you diagnose Polycythaemia Vera?

A

• After got diagnosis- think whats causing it
• Persistent increased Hb/hct>0.5
• 2 questions;
o Relative V absolute polycythemia
o Primary V 2ndry polycythaemia
• Spin blood sample- % of RBCs to plasma
• Detailed history & examination
• 1st line tests; FBC, Ferritin, Epo level, UE/LFT
• 2nd line tests;
o Epo elevated; CXR, ABG, USS abdomen
o Epo normal/ low; JAK2 mutation, bone marrow examination, EXON12 mutation

115
Q

What is Primary Polycythemia?

A

Polycythemia vera

116
Q

What is Secondary Polycythemia?

A
•	Central hypoxic process;
o	Chronic lung disease
o	Right-to-left shunts Heart Disease
o	Carbon monoxide poisoning
o	Smoker
o	High Altitude
•	Renal disease 
•	EPO production Tumours 
•	Drug associated
o	Treatment with androgen preparations 
o	Postrenal transplant erythrocytosis 
•	Congenital 
o	High oxygen-affinity haemoglobin 
o	Erythropoeitin recp-mediated 
•	Idiopathic erythrocytosis
117
Q

What mutation is present in Myeloproliferative Disorders ?

A

JAK2 Mutation in Myeloproliferative Disorders
• Occurs in JH2 domian (inactive)
• G- to- T mutation at nucleotide 1849
• Phenylalinine for valine at 617 in protein (V617F)
• Destroys BsaXI site
• Patients may be heterozygous/homozygous

Presence of JAK2 V617F mutation in peripheral blood DNA is diagnostic of a myeloproliferative disorder!

JAK Tyrosine Kinases
• JAK- signalling pathway for cytokine receps (GM-CSF, GCSF, EPO, TPO, SCF, interleukins)
• 4 known members of JAK family

The Test
• Allele specific DNA-based PCR
• Control PCR in same reac for normal gene
• Possible results;
o Normal control band only
o Mutant + control band
o No bands or mutant only= inadequate material

118
Q

What is the treatment and prognosis for

A
Treatment PV
•	Venesections (take a blood until RBC decreased to appropriate level) aim HCT <0.45
•	Aspirin 75mg daily 
Prognosis PV
•	Good- 15yr median survival 
•	Risk developing AML
•	Risk developing myelofibrosis
119
Q

What is Thrombocytosis?

A
  1. Primary Essential Thrombocytosis (ET) (bone marrow making too many platelets)- thrombosis risk
  2. Reactive thrombosis (in stress response platelets released- need to make sure pre-op level was normal); surgery, infection, inflamm, malignancy, iron deficiency, hyposplenism, haemolysis, drug induced (steroids, adrenaline, TPO mimetics), rebound post chemo
120
Q

What are the investigations for Thrombocytosis ?

A

Thrombocytosis Investigation
• Good history/ examination IMPORTANT (e.g. recent normal count prior to surgery- wait for levels to normalise)
• Persistent platelets >450 x 190/L

1st Line Investigations
•	FBC &amp; film 
•	Ferritin 
•	CRP
•	CXR
•	ESR 
If no known cause refer to haematology!
2nd Line Investigations 
•	JAK2
•	CALR
•	Bone marrow biopsy
•	Extensive search for 2ndry cause
121
Q

What mutations are found in Essential Thrombosis?

A

o JAK2 mutation- 50%
o CALR mutation- 45%
o Bone marrow consider

122
Q

What is the CALR Mutation ?

A

CALR Mutation
• Calreticulin mutation
• Cell signalling protein made in ER (endoplasmic Reticulin)
• Mutation EXON 9 of gene
• Found in myeloid progenitors in ET
• Mechanism of action unknown at present but may activate cell signal pathways
• Found in up to 90% JAK2 –ve ET

123
Q

What is the Treatment and prognosis for Essential Thrombosis (ET)?

A
•	Assess thrombotic risk; 
o	Age 
o	Hypertension
o	Diabetes
o	Platelet count >1500
o	History of thrombosis 
•	Antiplatelet treatment- aspirin 75mg daily 
•	Cytoreduction (only if High risk) 1 or more factors

Prognosis
• 20yr median survival
• Risk of AML or Myelofibrosis
• CALR mutated have lower thrombosis risk

124
Q

What is Cytoreduction?

A
  • Hydroxycarbamide
  • Interferon (give subcutaneously 1/wk)- molecular negativity
  • Anagrelide- interacts with megakaryocytes, reducing platelet produc
  • P32 (injection, lasts between 3-6mnths)
125
Q

What is Myelofibrosis?

A

• Presentation;
o Pancytopenia- all cells low (WBCs, RBCs & platelets)- often caused by bone marrow condition
o B symptoms- weight loss, fever & night sweats (associated with haematological malignancies)
o Massive splenomegaly
• Investiagtions
o FBC & blood film- tear drop shaped RBCs, &; will see erythroblast that still has its nucleus
o Haematinics

126
Q

What mutations are present in Myelofibrosis?

A
Diagnosis 
o	Blood film 
o	Bone marrow results 
o	JAK2 mutation 50%
o	CALR mutation 30%
127
Q

How do you measure Spleen Size?

A

• Mid clavicular line at bottom of ribs in top R hypochondrium- diagonally down to L as far as the tip of the spleen goes

128
Q

What are the causes of splenomegaly?

A
  • C- cancer
  • H- haematological- myelofibrosis, CML
  • I- infec- schistomiasis, malaria
  • C- congestion- liver disease/ portal
  • A- autoimm- haemolysis
  • G- glycogen storage disorders
  • O- other- amyloid etc
129
Q

What is the Treatment and prognosis for Myelofibrosis?

A

Treatment
• Supportive care
• JAK2 inhibitors
• Bone marrow transplant if patient young (50% mortality)

Prognosis
• Poor
• Median survival 5 yrs

130
Q

What is Chronic Myeloid Leukaemia ?

A
  • Pulripotent stem cell disorder
  • Defined by t(9;22) translocation, driven by BCR-ABL fusion tyrosine kinase
  • Chronic phase followed by acute transformation
  • Designer molecule treatment (imatinib/ gleevac) very successful (BCR-ABL mutations confer resistance to imatinib)
  • Rare- approx. 1 per 100,000 per yr
  • <10% aged <20yrs
  • Median age at diagnosis is 55-60 yrs
  • M:F ratio 1.5:1
  • Chronic=untreated natural history!
131
Q

What are the clinical features of Chronic Myeloid Leukaemia ?

A
  • Leucocytosis +++ (normal WC count 4-11, these patients have e.g. 300)
  • Leucoerythroblastic blood pic
  • Anaemia
  • Splenomegaly
132
Q

What are the symptoms of Chronic Myeloid Leukaemia ?

A
  • Abdominal discomfort; splenomegaly
  • Abdo pain; splenic infarction
  • Fatigue; anaemia, catabolic state
  • Venous occlusion; retinal vein, DVT, priapism
  • Gout; hyperuricaemia
133
Q

What is the Chromosomal abnormality in CML ?

A

Chromosomal abnormality in CML
• Philadelphia chromosome
• Part of chromosome 9 &a22 merge (9;22 translocation)
• BCR gene on chromosome 22 &; AbI gene on chromosome 9 = oncogene
• Can get diff break points where these are caused- normally p210 break point in CML patients
• Gleevac (imatinib mesylate)- tyrosine kinase inhibitor
• Designed to block bcr-abl tyrosine kinase active site
• Turns cells cycle off- cells differentiate & die
• Measure molecular response
• Imatinib & chronic myeloid leukaemia- designer molecular therapy, model & paradigm for other cancers, tyrosine kinase inhibitors increasingly used in solid tumours

134
Q

What is the treatment for CML?

A

CML Treatment before 2000
• Chronic phase (>90% patients presenting)- low dose oral cytotoxic drugs (busulphan, hydroxycarbamide)
• Interferon;
o Acute leukemic transformation/blast crisis (myeloid &lymphoid types)- intensive chemo, poor outcome

Imatinib Resistance
• Can get mutations to imatinib
• Activating loop mutations in BCR-ABL confer resistance & loss of disease control
• New TKI e.g. Nilotinib, Dasatinib helpful

135
Q

What are the two stages in B cell differentiation?

A

B-Cell Differentiation- Antigen Independent Stage
• Naïve B-cell = B-cell not exposed to an antigen (once exposed to antigen becomes memory or plasma cell that secretes antibodies specific to antigen bound)
• Antigen-independent B-cell differentiation forms the naïve B-cell- can respond to antigen &; express complete surface IgM, IgD &pan B-cell markers (CD 19, 20, 22, 40, 79a), complete receptors, CD23 &some express CD5
• Diff antigens for diff stages of B cell life- can yse this to differentiate &help diagnosis
• Naïve B-cells have rearranged but unmutated Ig gene
• Each B cell committed to a single light chain (kappa or lambada)

B-Cell Differentitaion- Antigen Dependent Stage
• Mature antigen-naïve B-cells can either be activated;
o Directly by antigens (apoptose or differentiate into short lived plasma cells with IgM antibody produc (primary immune response- no memory))
o Or with T-cells input
• Naïve B-cells that encountered antigen in paracortex (area of lymph node mostly consisting of T cells) or T cell zone, migrate to centre of primary follicle to form a germinal centre (GC)
• 3-10 naïve B-cells form each germinal centre (eventually contains 10,000-15,000 B-cells)
• 4 step process for naïve B-cell blasts to become either plasma or memory B-cells;
1. Proliferation
2. Immunoglobulin somatic hypermutation &class switch
3. Selection
4. Differentiation
• Antigen stimulation = B-cells differentiate into centroblasts = accumulate in dark zone of GC
• Centroblasts highly proliferative (cell cycle completed within 6-12hrs)
• Within centroblasts BCL-2 anti-apoptotic genes downregulated &; so proapoptotic molecules (e.g. CD95) only cells which generate highly specific receps to the antigen GC will survive
• Somatic hypermutation of centroblasts Ig V-region genes occurs within the GC- this increases intraclonal diversity
• Centroblasts mature = non-proliferating centrocytes (found in light zone of GC)
• Here heavy chain class switch occurs; alters Ig constant regions to IgG, IgA or IgE;
o If centrocytes Ig gene mutation results in lower affinity antibody for antigen in question it undergoes apoptosis
o If gene mutation results in increased affinity = can bind antigen (which up to now has been trapped by complenment receps on follicular dendritic cells)
• Centrocytes process antigen & present it to T cells which express CD40 ligand
• This binds to B-cell CD40 & rescues it form apoptosis
• Need to inactivate BCL6 (is regulator) for post GC differentiation (into memory/ plasma cells)
o Numerous mechanisms which inactivate BCL6 e.g. CD40-CD40 ligand interaction; stimulates centrocyte expression of IRFA (represses BCL6)
o BCL6 has an inhibitory effect on BLIMP1; so when BCL6 downregulated BLIMP1 is upregulated this represses = PAX5 enabling plasma cell differentiation
o BLIMP1 also upregulates XBP1- helps regulate plasma cells in their secretory phenotype
Another important step in terminal differentiation in proliferating tissue is downregulation of myc gene- important in Burkitt lymphoma pathogenesis

136
Q

What is the Plasma Cell Differentiation Pathway ?

A

Check up: loads of cells involved in production of the plasma blast which later becomes the plasma cell.
involves:
• IRF4= interferon regulatory factor 4
• BLIMP1= B-lymphocyte induced maturation protein
• XBP1= X-box binding protein 1
• PAX5- paired box protein 5
• MYC= regulatory gene- codes for a transcription factor
• Regulatory mechanisms- can make neoplastic plasma cells

137
Q

What antibodies are produced in myeloma?

A

Important to know about in myeloma- churns out 1 type of antibody (paraprotein)

138
Q

What are Immunoglobulins?

A

• Plasma cells make glycoprotein molecules in response to an immunogen
• Composed of 4 polypeptide chains (2 light &; 2 heavy chains- held together by disulphide bonds)
• Each chain has 1 variable & 1 constant region- polyclonal Igs made normally
• Classified according to amino acid sequences in constant region of;
o Heavy chains- IgG, IgM, IgA, IgD, IgE
o Light chains- kappa or lambada

139
Q

What is Protein Electrophoresis? What 5 types of protein are identifies using this technique?

A
•	Lab technique- serum placed in a gel &amp;exposed to elec current 
•	5 major fractions normally identified;
o	Serum albumin
o	Alpha-1 globulins
o	Alpha-2 globulins
o	Beta globulins
o	Immunoglobulins
140
Q

What is Immunofixation?

A
  • Enables detection &; ID. of monoclonal immunoglobulins
  • Can detect the one type of immunoglobulin the myeloma makes
  • Perform when ‘M-spike’ seen on electrophoresis
  • Serum/urine placed on a gel the apply electric current to separate proteins
  • Anti-immunoglobulin antisera added to each migration lane
  • If immunoglobulin present- complex precipitated
141
Q

What is a Plasma Cell Neoplasm?

A

Too many plasma cells & not many of other cells

142
Q

What is a Myeloma ?

A
  • Incurable malignant disorder of clonal plasma cells
  • 60-70million ppl/yr in UK
  • Median age presentation= 70 yrs
  • Higher incidence in Afro-Caribbean ethnic groups compared with Caucasians
  • Is preceded by asymptomatic MGUS (monoclonal gammopathy of undetermined significance- abnormal protein made by plasma cells- cause no probs) in all patients
  • Spectrum of plasma cell dyscrasias;
  • MGRS- linked with renal
  • Solitary plasmocytoma- lump of plasma cells in ribs, back, vertebrae, hips- disease associated with plasma cells
143
Q

What is the Myeloma- IMWG Diagnostic Criteria ?

A

• Clonal BM (bone marrow) plasma cells >10% or biopsy-proven bony or extramedullary plasmacytoma (plasma cell tumours outside bone marrow) AND any one or more of:

o CRAB features (symptoms that show myeloma active- end organ damage);
(symptoms; Calcium elevated, Renal failure, Anaemia, Bone lesions)
 C- hypercalacemia (>2.75mmol/L)
 R- renal insufficiency (creat clearance <40ml/min or serum >177micromol/L)
 A- anaemia (Hb<100g/L)
 B- bone lesions (1 or more osteolytic lesions on skeletal radiography, CT, or PET/CT)

oMDEs (myeloma defining events)
 > 60% clonal plasma cells on BM biopsy
 SFLC (serum free light chain- churn out immunoglobulin light chains circulating in serum) ratio >100mg/L provided the absolute level of the involved LC is >100mg/L (some just give out either kappa or lambda light chains- monoclonal)
 >1 focal lesion on MRI measuring >5mm

  • Pepperpot skull
  • Implications for ‘asymptomatic/ smouldering’ myeloma cases as picking up more end organ damage- more inclined to treat
144
Q

How does Myeloma effect the Kidneys ?

A

• 20-25% patients- renal insufficiency at diagnosis
• 50%- renal insufficiency at some point during disease course
• 50% have persistent renal impairment despite therapy
• 2-12% need RRT
External factors;
• Abnormal paraprotein gets stuck in kidney
• Myeloma prothrombotic- can get renal vein thrombosis
• When treat high calcium- drugs nephrotoxic
• People become dehydrated in myeloma (inadequate intake)
• NSAIDs for bone aches are nephrotoxic
• Contrast given with CT can upset kidneys
• Abnormal protein in blood increases it’s viscosity & clogs it up

145
Q

What are common symptoms of myeloma?

A
Common Clinical Features!!!
•	Fatigue &amp; malaise 
•	Bone pain- particularly back &amp;; rib pain 
•	Infective episodes (as normal antibosied not being made properly)- particularly pneumococcal chect infecs 2ndry to failure of normal Igs produc 
•	Hypercalcemia symptoms;
o	Constipation
o	Anorexia
o	Confusion
o	Polyuria 
•	Renal impairment symptoms;
o	Confusion 
o	Oligouria/ anuria
146
Q

What is Monoclonal Gammopath of Uncertain Significance ?

A

Non-cancerous condition. People with MGUS make an abnormal protein, called a paraprotein or M-protein which is usually asymptomatic.

People with MGUS do not usually need any treatment. But there is a small risk of MGUS developing into a blood cancer, such as myeloma

  • 3.2% ppl >50yrs, 5.3% >70yrs, 8.9% >80yrs
  • M>F
147
Q

What is the MGUS- IMWG Diagnostic Criteria?

A

• MGUC- IMWG Diagnostic Criteria;
o Serum M-protien <30g/L
o <10% clonal plasma cells in bone marrow
o Absence of end-organ damage (CRAB) or myeloma-related events (MREs)

148
Q

What are the Risks of MSUS Progression

A

• MGUS- Risks of Progression;
o Approx 1% per yr
o Majority progress to myeloma (27% in 1 series) but can progress to other conditions
o Others progressed to;
 Waldenstrom’s macroglobulinaemia (type of myeloma)
 Primary AL amyloidsis
 Lymphoprolifertive disorders
o High vs low M-protien (15g/L)
o IgA/IgM have higher risk of progression vs IgG PP
o Abnormal SFLC ratio vs normal

149
Q

What is AL Amyloidosis ?

A
  • Neoplastic plasma cells make small amount of abnormal immunoglobulin- fibrils deposit in kidney
  • Amyloid light chain (AL) amyloidosis
  • 600 new cases/ yr in UK
  • Light chain fragments misfold & self-aggregate = form beta-pleated fibrils
  • 1.5% of native kidney biopsies
  • Nephrotic-range proteinuria; mainly albumin, small monoclonal light chain component
  • Cardiac (heart failure) & liver involvement (hepatomegaly) in 30%
  • Peripheral neuropathy in 10%
  • ESRF (end-stage renal failure) in 40%
150
Q

What is the treatment of AL Amyloidosis ?

A

Treatment same as for myeloma- aim to ‘switch off’ light chain production by plasma cells, difficult to remove already deposited proteins = signif morbidity & mortality

151
Q

What is Non-Hodgkins Lymphoma ?

A

Malignant lymphoma
T Cell:
B Cell: High grade(burritos/diffuse large B cell lymphoma) and Low grade (Follicular lymphoma. hairy cell, mantle cell, marginal zone)
high grade- acute onset, several LN groups affected, systematic symptoms, abnormal bloods, curative but poor prognosis if relapse
:Low grade: onset maths-years, small LN sites, clinically well normal bloods, non curative but often relapse and remission throughout course.

  • Low grade- lymph node slowly growing but otherwise well
  • Treat high grade by chemo
  • Low grade- slow rate of prolif- so not as chemo sensitive
152
Q

What to ask in a lump history?

A
  • Nature of lump- size, rate of changes, tenderness, skin changes, history of trauma
  • Additional lumps/ lesions elsewhere (particularly breast lumps)
  • History of weight loss and/ or night sweats
  • History of breathlessness/ cough/ haeamoptysis
  • Past medical history- particularly previous malignancies
  • Social history- smoking history particularly
  • Fam history- of bone marrow disorders/ malignancies
153
Q

What are the possible causes of a neck mass?

A
  • MALIGNANT- lymphoma, chronic lymphocytic leukaemia (CLL), metastatic cancer of lung/ breast/ cervix to lymph node
  • NON-MALIGNANAT- infective (bacterial, viral, mycobacterial), inflammatory (sarcoidosis), lipoma, fibroma, haemangioma

Further investigations?
• Bloods; FBC, U&Es, LFTs, CA2+, LDH, immunoglobulins & protein electrophoresis
• Imaging; chest x-ray, ultrasound scan of neck lump, fine needle aspirate &/or core needle biopsy of lymph node

154
Q

What is a Follicular Lymphoma ?

A
  • Neoplastic disorder of lymphoid tissue
  • Type of non-Hodgkin lymphoma characterised by slowly enlarging lymph nodes
  • Accounts for approx. 15% of all non-Hodgkin lymphoma diagnoses
  • Incidence rises with age
  • M=F
  • Median survival 8-10 years, five-year overall survival 72-77%
155
Q

What genetic abnormality causes Follicular Lymphoma ?

A
  • Brings bcl2 protooncogene under influence of immunoglobulin heavy-chain gene leading to bcl-2 protein over-expression
  • This in turn confers a survival adv to neoplastic lymphoid cells by inhibiting apoptosis
156
Q

What is the Follicular International Prognostic Index (FLIPI) ?

A

• Follicular International Prognostic Index (FLIPI) can be used to prognosticate:
o age >60 years
o Ann Arbor stage III or IV- widespread
o LDH above the limit of normal at diagnosis
o Hb<120g/L (anaemic at diagnosis)
o presence of more than four nodal sites of disease
• If patient has 4 or more prognostic factors, 10 yr survival rate is 36% compared with 71% for those with 1 or none

157
Q

What is Hodgkin Lymphoma ?

A

• Malignant disorder of neoplastic lymphocytes characterised by Hodgkin Reed-Sternberg (HRS) cells in a cellular infiltrate of non-malignant inflammatory cells eg: eosinophils
• Annual UK incidence is 2.7/100,000
• Associations with Ebstein-Barr virus & HIV
• Classification;
o Nodular lymphocyte predominant
o Classical;
 Nodular sclerosis
 Mixed cellularity
 Mixed lymphocyte-rich
 Lymphocyte depleted
• Managed- chemotherapy (e.g. ABVD), radiotherapy
• Doses/ number of courses depends on stage;
o Early stage;
 Favourable (ESR <50 & no B syx, ESR<30 and B syx, no large mediastinal LN, age <50 years, 1-3 LN sites involved)
 Unfavourable
o Advanced stage
• Prognosis- high proportion cured (86% 5 yr survival)
• Long term therapy effects important;
o Increased mortality is still seen at >20 years post therapy
o Pulmonary toxicity
o Cardiovascular disease
o Secondary malignancies
• Balance between curing &long term side effects

158
Q

How do you distinguish Malignant lymphoma from Hodgkins disease?

A

Malignant lymphoma distinguished from Hodgkins disease by absence of binucleate giant cells

159
Q

What is Chronic Lymphocytic Leukaemia?

A
  • Malignant disorder of mature B-cells
  • Most common type of leukaemia in UK (1% of all cancers in the UK)
  • Incidence of approx. 5 per 100,000/yr in UK
  • Presentation ranges from incidental findings of lymphocytosis (too many normal lymphocytes n blood- cells look squashed & smeared- CLL sign) to wide spread lymphadenopathy, splenomegaly, bone marrow failure &; systemic symptoms.
160
Q

What is the staging for CLL?

A

Binet System
A: Hb and platlets over 100 <3 areas of lymphadenopathy (>10yrs survival)
B: Same as A but > 3 areas of lymphadenopathy (8yrs survival)
C: Platlets and HB below 100 and any number of lymphadenopathy (6.5yrs survival)

161
Q

Possible complications of CLL

A

transformation to high-grade lymphoma (DLBCL) = Richter’s transformation:
• Rare complication of CLL - <10% of cases
• Acute onset of symptoms
• Poor prognosis = median survival <9 months even with intensive chemotherapy
• Progressed to a higher grade lymphoma

Other possible complications of CLL:
• Recurrent infecs 2ndry to reduced immunoglobulin produc- mature B cells not making normal antibodies
• Autoimmune phenomenon:
o ITP- immune thrombocytopeia
o Haemolytic anaemia
o Pure red cell aplasia
o Autoimmune neutropenia- make antibodies that destroy cells

162
Q

RBCs at birth and childhood

A

o Initially make RBCs in yolk sac, liver kicks in in 2 ½ mnth then spleen kicks in
o At 5 mnths liver & spleen at max capacity to make RBCs
o Bone marrow joins in at 4 mnths
o Then spleen stops joing in, then liver (bone marrow compentant)
o Site of haematopoiesis varies
o Hb switching
o At birth 55-65% is HbF
o Diff in RBC struc & metabolism
o Larger RBcs
o Higher haematocrit (ratio of RBCs to total blood vol)
o Rapidly reach similar values to adults (12-16 wks)- unless you have a condition e.g. Downs syndrome

163
Q

What is Haemoglobin Switching ?

A
Haemoglobin Switching 
•	Chromosome 16 = 2 alpha chains: zeta to alpha (ζ → α)
•	Chromosome 11 = 2 beta chains: epsilon to gamma to beta to delta (e → γ → β → δ)
•	Body decides e.g. chromosome uses epsilon to make all it’s beta, then gets sick of this and goes to gamma (can’t go backwards to using epsilon)
o	4-14/ 40 gestation:
	ζ2e2 = Hb Gower-1
	ζ2γ2 = Hb Portland 
	α2e2 = Hb Gower-2
o	>14/40 gestation (next gestation):  
	α2 γ2 = Hb F
o	Neonatal stage (adult Hb being made): 
	α2β2 = Hb A 
	α2δ2 = Hb A2
164
Q

Child v Adult: White Blood Cells

A

• Similar numbers- higher lymphocyte counts
• Immune system is a work in progress;
o IgG crosses placenta
o IgM stays in mother (M for mother)
o Breast milk has IgA, IgD, IgE, IgG, IgM = get range of antibodies- passive immunity
o Start producing antibodies at 2-3/12 mnths
o (at 12mnths can start making antibodies for most things)
o Can make satisfactory immune response by 6/12

165
Q

Child v Adult: Platelets

A

• Reach adult numbers by 18/40 gestation
• Initially larger (so even if have less can stiff provide response), by birth slim down to adult size
• Functionally diff at birth;
o Hyporesponsive to certain agonists
o Hyperresponsive to vWF
• Diffs balance out

166
Q

Child v Adult: Haemostasis

A
  • Present but imperfect at birth
  • Coag proteins don’t cross placenta effectively- don’t want them to clogg umbilical cord up
  • Only fibrinogen, FV, FVIII, FXIII are present at normal birth (fibrin holds them together)
  • Most haemostatic paramenters reach adult values by 6mnths
  • Free probs clinically unless ill or preterm
167
Q

What is Vit K what is its important in the newborn?

A
  • FII, FVII, FIX, FX, protein C, protein S are vit K dependant
  • Placental gradient means foetal vit k is 10% of that of mothers
  • Haemorrhagic disease of the new born prevented by giving routine neonatal vit K injecs
  • Exacerbated if mum is on anti-convulsants- mum needs oral vit K
  • Warfarin is teratogenic
168
Q

What are congenital blood disorders?

A
CONGENITAL 
•	Hb synthesis prob- haemoglobinopathy 
•	Bone marrow failure syndromes
•	Bone marrow infiltaration 
•	Peripheral destruction (making something tha s being destroyed, or are losing it through bleeding) 
•	Blood loss
169
Q

What can you give for sickle cell treatment?

A
  • Sickle cell- give hydroxycarbomide- shifts them back to make HbF, gamma gene is healthy (unlike beta gene) so have less crisis
  • Can’t go biologically backwards, but medication can make them go back to gamma.
170
Q

What is peripheral destruction?

A

Peripheral Destruction
• Autoimmune haemolysis (RBCs more susceptible to haemolysis)
• Haemolytic disease of new born- antibodies recognising something from somebody else that they recognise as foreign
• Rh/ABO or other incompatibility
• Infec
• Memb defect: hereditary spherocytosis (RBCs sphere shaped like baubles- they pop so become hypoxic)
• Enzyme defect: G6PD deficiency, PK deficiency

171
Q

What types of blood loss can happen in the womb?

A
  • Twin to twin transfusion- one twin takes all blood available in womb
  • Red baby- high level of Hb- blood can’t move out easily & it’s like sludge- blood so thick will not go around vessels well (need to dilute/drain them)
  • Fetomateral haemorrhage
172
Q

What is Acquired Childhood Anaemia ?

A

• Iron deficient
• Acquired;
o Nutritional deficiency: iron, B12, folate
o Less b12 & folate deficiency seen if mum is vegan perhaps
o Bone marrow failure
o Bone marrow infiltration
o Peripheral destruction (when slightly older): haemolysis
o Blood loss

173
Q

What causes Bleeding &Bruising in children?

A

• Congenital;
o Platelet probs- diagnose through looking at blood film
o Clotting factor prob
o Connective tissue disorder e.g. Ehlers Danlos
• Acquired;
o Trauma: accidental, non-accidental
o Tumour
o Infec: acute e.g. meningococcus (see spots coming up- don’t blanch with glass), chronic e.g. HIV
o Immune disorder: primary- immune thrombocytopenia, TTP, secondary- SLE (lupus), ALPS
• Bone marrow failure
• Drug related

174
Q

What is Haemoglobin ?

A
•	Tetramer of globin chains, each non-covalently bound to a Haem 
•	All normal haemoglobins have 2 alpha &amp; 2 non-alpha chains 
•	Hb A tetramer;
o	Globin 
Protects haem from oxidation 
o	Renders molecule soluble 
o	Permits variation in oxygen affinity 
•	Genetic control of globin chains;
o	Diff between chains in each parent 
o	2 loci for alpha gene on each chrom
o	But 1 loci on each gene for beta chains
•	Normal adult Hb;
o	Hb-A (2α/ 2β)- >95
o	Hb-A2 (2α/ 2δ)- <3.5%
o	Hb-F (2α/2ϒ)- <1.0
•	Haematopoesis moves from yolk sac (up to 10wks) to liver &amp;  spleen after about 12wks
175
Q

What are common Haemoglobinopathies?

A

• Changes in globin genes/ their expression leads to disease (how structurally varies- struc of Hb chain)
• STRUCTURAL Hb variants
o e.g. Hb S (sickle), C, D, E etc
o Usually a single base substitution in globin gene = altered struc/ function
• THALASSEMIAS (alpha or beta)-
o Change in globin gene expression leads to reduced synthesis rate of NORMAL globin chains.
o Pathology due to IMBALANCE of alpha &; beta chain produc e.g. reduction of normal globin chains(free globin chains damage red cell memb)

176
Q

What is Physiological Anaemia &; Macrocytosis in pregnancy?

A
  • Plasma vol expands in preg by 50%
  • Red cell mass expands by 25%
  • Haemodilution occurs maximally at 32wks
  • CDC define anaemia as Hb <11g/dL 1st & 3rd trimester &; 10.5 in 2nd trimester (ananemia must be investigated- iron deficiency moast common)
  • MCV increases physiologically in preg; increased iron requirements usually results in considerable mobilisation of Fe stores, folic acid requirements also increased
177
Q

What is Leukocytosis ?

A
  • Placenta makes chemical that makes bone marrow make more white cells
  • WBCs increase in preg
  • Mainly a neutrophilia, rising form 2nd month to peak range of 9-15 in 2nd-3rd trimester
  • L shift may also be seen (myelocytes/ metamyelocytes)
178
Q

What is Gestational Thrombocytopenia?

A
  • Platelet count drops (usually >70x109/l)
  • Falls after 20wks & thrombocytopenia most marked in late preg
  • No pathological signif for mother or fetus
  • Recovers rapidly following delivery
  • Main issue in management is differentiation from other causes
179
Q

What is Pregnancy associated Thromboctopenia ?

A
•	Produc failure- 
o	severe folate deficiency
•	Consumptive- 
o	gestational
o	pre-eclampsia &amp; HELLP syndrome, 
o	AFLP (acute fatty liver pregnancy- can drop platelets) 
o	DIC e.g. abruption
o	TTP/HUS (haemolytic
180
Q

What is Coincidental Thrombocytopenia ?

A
Coincidental Thrombocytopenia (May not be just due to preg)
•	Produc failure- 
o	bone marrow infiltration/ hypoplasia 
•	Consumptive- 
o	ITP: primary/ 2ndry
o	Viral (HIV, EBV)
o	Sepsis
o	Type 2B vWD
o	Hypersplenism
•	Congenital
181
Q

Coagulation in Pregnancy

A

Coagulation: Pregnancy is a Prothrombotic State
• Evidence of platelet activation
• Increase in many procoag factors
• Reduction in some natural anticoags
• Reduc in finbinolysis
• Rise in markers of thrombin generation
• Coag factors;
o Marked increase in plasma fibrinogen & factor VII
o Increase also in factor V, VIII, X, XII
o Greater increase in vWF than factor VIII (2 fold in late preg)
o Minimal increase in FIX & small decrease in FXI
o Initial increase in FXIII followed by reduction to approx. 50% of non-preg value

182
Q

How are Haemoglobinopathies diagnosed?

A

• Structural Hb variants usually detected by abnormal mobility on Hb electrophoresis
• Thalassaemias have normal Hb Electrophoresis but small pale RBCs (hypochromic/microcytic)- resembling iron deficiency
• Tests rely on differentiating normal & abnormal proteins (first 4 tests on list) &; specialised testing done in certain situations;
o FBC/ film
o Hb electrophoresis
o Isoelectric focusing
o High performance liquid chromatography (HPLC)
o Heat stability, isopropanol (unstable Hb’s)
o Oxygen dissociation curve (p50, high affinity)
o DNA analysis (genetic counseling, prenatal Dx)
o Mass spectometry
o Kleihauer testing, Supravital staining, Sickle solubility

183
Q

What is sickle cell disease?

A

• SS Hb had excess +ve charge compared to healthy Hb
• Valine substituted for glutamine at position 6 of β- globin gene
• Sickle Hb (Hb-S) polymerises at low oxygen tensions to form long fibrils (tactoids)- distort red cell memb & make classical sickle shape
• Sickled RBCs have a short lifespan in blood- haemolytic anaemia
• Polymerisation reduced if other haemoglobins present in red cell e.g. Hb-F in fetus or neonate
• Haemolysis-associated haemostatic activation
o Sickle cell interact with vasc endothelium= adhesion molecules formed = other normal cells adhere to vessel causing damage= thrombotic state
o Intravasc hemolysis releases Hb into plasma = quenches NO & makes reactive oxygen species
o Arginase I released form RBC during hemolysis &metabolizes arginine (substrate for NO synthesis) further impariring NO homeostasis
o NO depletion associated with pathological platelet activation & tissue factor expression
o Haemolysis &; splenectomy also associated with phosphatidyserine exposure on RBCs which can activate tissue factor &; form platform for coag.

Electron Microscopy of Sickle Cells Showing Distortion by Fibrils of Polymerised Hb-S
• Sickle cells start polymerising
• Can block capillaries pain &lack of O2 to tissues (sickle cell crisis)
Life of sickle RBC 10-20 days (normal RBC lifespan is 120 days)

184
Q

What is Sickle Cell Trait ?

A

HETEROZYGOUS (Hb A/S)
• Blood count: normal
• Hb Electrophoresis: Hb-S 45%, Hb-A 55%
• Clinical pic: no probs except when extreme hypoxia/ dehydration (e.g. very bad anesthetia, flying unpressurised military aircraft)
• Minor complications- minor infrction of renal pili haematuria
• Higher risk of renal carcinoma in those with sickle cell trait
• Sickle cell results in anaemia
• Hb electrophoresis shows predominantly S
HOMOZYGOTE (Hb S/S)
• Blood count: anaemia (Hb 6-8g/dl)
• Blood film: sickle cells
• Hb Electrophoresis: Hb-S >95%, Hb-A 0%
• Freq: 1 in 200 Afrocaribeans, 1 in 60 West Africans (around 12,000 pateints in UK, compare to 5,000 heamophiliacs)

185
Q

What are Acute Complications of Sickle cell disease:?

A
  • Vaso-occlusive crisis - hands and feet (dactylitis), chest syndrome, abdominal pain (mesenteric), bones (long bones, ribs, spine), brain, priapism
  • Septicaemia- as spleen gets gradually infarcted
  • Aplastic crisis- can be due to viruses; interrupts RBCs for a wk so if RBC life only 10-20 dyas Hb drops anaemia can die
  • Sequestration crisis (spleen, liver)- organs enlarge
186
Q

What are Chronic Complications of Sickle cell disease:?

A
  • Hyposplenism- due to infarction and atrophy of spleen.
  • Renal disease: medullary infarction with papillary necrosis (renal tissue infarcted away need dialysis). Tubular damage - can’t concentrate urine (bed-wetting at night). Glomerular – chronic renal failure/dialysis
  • Avascular necrosis (AVN) –femoral/humeral heads (1 blood supply)
  • Leg ulcers, osteomyelitis (high bone infecs), gall stones, retinopathy, cardiac, respiratory (resp probs e.g. pul hypertension)
  • Mortality: ~ 0.5% per annum in UK
187
Q

What is the Treatment of Sickle Cell Disease ?

A

Treatment of Sickle Cell Disease
• Penicillin etc from 6mnths (neonatal screening)
• Acute crisis
o Patients need to come into hospital within 3 hrs
o Vaso-occlusive- analgesia (usually opiates), hydration (to maintain red cell water), treatment of precipitants
o Priapism- education. Acute (minor/major, intracorporeal phenylephrine), Recurrent etilefrine
• Respiratory
o ECHO screening
o O2 sats
• In clinic try to prevent longer term organ damage
• E.g. ACE inhibitors may slow down progressive renal impairment
• Transfusion
o Top up: splenic sequestrain, aplastic crisis, pre-operative, acute chest crisi (usually when Hb <5g/dl)
o Exchange: acute chest crisis, acute stroke, pre-operative (chronic organ damage)
o Regular exchange: primary & 2ndry stroke prevention
• Transfusion may not be helpful for painful crisis (don’t just transfuse for pain)
• How low is anaemic- if Hb <50- get cardiac strain on heart- most people will transfuse at this point
• If top up above baseline may induce a stroke
• Can screen middle cerebral artery in kids- if flow rate exceeds certain rate indicates stroke- can prevent
• Hydroxycarbamide;
o Slow cell cycle down (RNA inhibitor)
o Increase HbF which decreases severity (time delay to polymerisation, reduced adhesion to endothelium, enhances NO)
o Consider if >3 admissions with painful crisis in 12 months or 2 chest crisis
• Acts as an anti-inflamm
• Transcriptional Doppler (annual from 3 yrs), STOP trial
• AVN (avascular necrosis)-MRI, may need joint replacement
• Perioperative care- transfusion, avoid local tourniquet
• Cholecystectomy (remove gall bladder) for symptomatic biliary disease
• Renal disease- U&E, BP, Ix haematuria (role of ACE inhibitors for proteinuria)
• Ophthalmic- annual review
• Curative
o Bone marrow transplant form normal donor (usually form sibling)
o Gene therapy (intro normal genes into cells in place of missing ones to correct genetic disorders)
• Prevention
o Genetic counselling/ prenatal diagnosis
o Avoiding precipitants

188
Q

What are Other Causes of Sickle Cell Disease ?

A
  • Co-inheritance of βs and another β chain abnormality
  • SS- sickle cell anaemia
  • SC sickle cells disease tend to have better survival than SS sickle cell disease (behave slightly diff)
  • SC disease- fewer crisis, ↑ risk AVN (avascular necrosis) & retinopathy
  • S/O-Arab (severe)
  • S/β- thalassaemia, S/Lepore, S/D-punjab (moderate)
  • S/HPFH, S/δβ0, S/E (mild)
189
Q

What is Thalasaemia?

A
  • Most common single gene disorders
  • Highly heterogenous- globin gene long- can have many point mutations
  • Alpha thalassemia- beta precipitating & damaging alpha
  • Divided into α, β, δβ and γδβ according to which globin chain is reduced (HPFH)
  • In some, no globin chain is produced (e.g. α0), in others they are produced at a reduced rate (e.g. α+)
190
Q

What is Alpha Thalasaemia?

A
  • We have 4 alpha genes
  • Fetal Hb has alpha chains in it
  • Most serious forms restricted to SE Asia & some Mediterranean islands
  • Both Hb A & F have a chains
  • Hb Barts (homozygous inheritance of a0)- hydrops fetalis
  • Hb Barts- still birth/ only survive for few wks
  • Hb H disease- gene defect on 3 out of 4; can still make enough Hb to survive, moderate
  • α-trait
  • Can’t detect alpha-trait- might have low MCV & MCH (small pale RBCS)- can detect with genotyping (but usually not necessary)
191
Q

What is Beta Thalassaemia?

A

• Reduced rate of beta-globin chains produc (pathology caused by excess alpha chains)
• Many molecular variants arising in diff locations worldwide
• Carriers (Thalassaemia Minor)- clinically normal;
o Blood pic resembles iron deficiency (small, pale red cells)
o Total Hb level normal/ slightly reduced
o Hb-A2 level >3.5%
o ‘Target cells’ on film
o No clinical probs
• Homozygotes (Thalassaemia Major)- have severe disease which is usually fatal if untreated (produce little, if any, Hb-A & die of severe anaemia)

192
Q

What is What is Beta Thalassaemia??

A

• What is Beta Thalassaemia?Can survive first few yrs of life without transfusion- don’t need regular transfusions in 1st 3-5yrs of life
• Can be alpha or beta chains (clinical pic & genetic basis can be highly variable)
• All should be genotyped?= severity of phenotype (see if thriving or need transfusion)
• When to transfuse/chelate?
• Role of splenectomy? Transplantation?
• Probs they can suffer from (as RBCS BROKEN DOWN);
o Pulmonary hypertension- ECHO
o Extrameduallary haematopoesis- other organs try making RBCs- enlarge
o Bone changes & osteoporosis
o Iron overload deposits in endocrine- endocrine & fertility (DM, Hypothyroid)
o Leg ulcers

193
Q

What is Beta Thalassaemia Major?

A

• Apparent after 6 mnths born (foetuses don’t have beta chains)
• Present with very severe anaemia at 1-2 yrs old
• Blood film very abnormal with lots of nucleated red cells
• Clinical features due to severe anaemia & attempt to make more red cells in marrow to compensate
• Pathology of beta thalassaemia major;
o Alpha chain excess= ineffective erythropoiesis (red cells die in marrow), shortened RBC lifespan (heamolysis= anaemia
o Increased marrow activity= skel deformity (stunted growth), increased iron absorption & organ damage (exacerbated by blood transfusion), protein malnutrition
o (Transfusion helps stop skeletal abnormalities)
o Enlarged & overactive spleen = pooling of RBCs in spleen (increased anaemia), increased transfusion requirement
• Short stature & distorted limb growth due to premature closure of epiphyses in long bones
• Enlarged liver & spleen (‘extramedullary haemopoiesis’)
• Thalassaemic facies (enlargement of cheek bones & foehead, bone marrow cavity expands &; cortex of bone thinner);
o Maxillary hypertrophy
o Abnormal dentition
o Frontal bossing due to expanded bone marrow
• X-ray showing ‘hair on end’ skull due to widening of diplopic cavities by marrow expansion

194
Q

What is the Treatment of Beta Thalassaemia Major ?

A

Treatment of Beta Thalassaemia Major
• Transfusion:
o To maintain mean Hb 120g/dl (pre-transfusion Hb 9.5-1.0)
o Suppresses marrow red cell produc & prevents skeletal deformity & liver/spleen enlargement
o 3-4 wkly transfusions from 1st yr of life
• Unfortunatley
o Each unit of red cells contains 200-250mg Iron &body has no excretory mechanism for iron
o By 10-12 yrs of age- severe iron overload & toxicity:
 Gonads/hypothalamus (failure of puberty, growth failure)
 Pancreas (diabetes)
 Heart (dilated cardiomyopathy &; heart failure)
 Liver (cirrhosis)

• To prevent death from iron overload- start patients on iron chelation therapy from 2nd yr of life to promote iron excretion in urine & faces;
o Desferrioxamine 8-12 hrly subcutaneous infucion via syringe pump as home-treatment on at least 5 nights a wk (prevents iron accumul)
o New Oral Iron Chelators- Deferiprone & Deferasirox
• Target ferritin around 1000-1500µg/L

Beta Thalassaemia Major- Prevention Better than Cure
• Genetic counselling- compulsory in some countries- led to dramatic fall in incidence
• Antenatal screening (routinely offered in UK)- high risk pregnancies can opt for prenatal diagnosis (chorionic villus biopsy &; genetic diagnosis at 8-12wks), termination of affected pregnancies
• Neonatal screening- must be sensitive to cultural &religious issues in a diverse pop

New Horizons in Treatment of Beta Thalassaemia Major
• Bone marrow transplant
o from a tissue-type matched donor (usually a sibling)
o 60-80% cure-rate if done in first few years of life but much less
o successful and high mortality if patient already has iron overload
o only 1 in 4 sibs will be a match
• Gene therapy (for the future)

195
Q

What is Transfusion Therapy & Monitoring Iron Loading in Thalassaemia ?

A

• RBC covered in proteins- match
• 1 blood unit contains 200-250mg iron (given this much in a transfusion)
• But you only need 0.47mg iron/mL in whole blood (so after transfusion would get an iron overload which is why you need chelation therapy alongside transfusion)
• ‘Pure’ red cells 1.16mg iron/mL
• A 60kg thalassaemia patient receiving 45 units of blood annually has transfusional iron intake of 9g iron/yr (0.4mg iron/kg body wt/day)
• Also up to 4mg/ day may be absorbed form gut- up to 1.5g iron/yr
• Other complications of transfusions include;
o Transmission of infec
o Allo-immunisation

Iron overload can= death

196
Q

What is the Treatment for Thalassaemia Major ?

A

Treatment for Thalassaemia Major
• Monitoring Chelation- ferratin (acute phase), liver biopsy, MRI T2 (MRI is gold standard)
• Infection- decreased CD4/8 ratio & defective neutrophil chemotaxis, increased virulence with excess iron (yersina &; DFO), line infecs, transfusion transmitted infec
• Endocrine complications- growth &development, glucose intolerance screening, hypothyroidism & hypoparathyroidism
• Liver disease- LFT, ferritin, Hep Serology, MRI. Hepatologist if established in disease
• Bone growth affected- timely transfusion, no over-chelation, diet, hormone replacement of hypogonadism, monitoring for OP, treatment with bisphosphates
• Fertility- most require IVF (once ovaries stim work well), can lead to arrhythmias (heart already stressed), good chelation, fertility clinic advice, may need induction. Cardiac, thyroid, diabetic and bone assessments. Obstetric supervision
• Psycho-socail issues (important for compliance)- Interpreters, written information, involvement in decisions (improves compliance), regular child development review (may include clinical psychologist and social worker)
• Importance of an MDT approach
• Care networks- annual review at a specialist centre
• Prognosis- most patients now live into their 4th & 5th decades of life although many current patients still have high iron levels & there are signif quality of life issues

197
Q

What is Virchows Triad ?

A

Factors that contribute to thrombogenesis (pathological clot in blood vessel);
• Alterations in blood flow producing stasis
• Damage to vascular endothelium
• Changes in blood constituents resulting in hypercoagulability

198
Q

What is Arterial Thrombosis?

A
  • Primary pathological abnormality usually atherosclerosis of vessel wall
  • And atheromatous plaques ruptures = endothelial injury= expresses more pro-coag proteins on surface (e.g. tissue factor)
  • Platelet aggregation &PLATELET thrombi important role in final vessel occlusion
  • Risk factors; smoking, hypertension, hypercholesterolemia, diabetes, fam history, obesity, physical inactivity, age, male sex.
199
Q

What is Venous Thrombosis?

A

athogenesis may involve;
o Venous stasis (e.g. in preg)
o Hypercoagulable state (heritable/ acquired) e.g. in preg blood becomes thicker in prep for delivery
• Thrombin mostly composed of FIBRIN (lesser role for platelet accumul &aggregation)
• VTE freq unrecognised- 80% of DVT clinically silent (danger- piece can break off & break in lung resp failure, small Pes can cause haemoptysis)

200
Q

What is the Epidemiology of VTE ?

A
  • Incidence; 1 per 1000pa
  • May present as sudden death (up to 20% of PE present as sudden death)
  • 30% develop deep recurrent VTE in 10yrs
  • 28% develop post thrombotic syndrome
  • Mortality of promptly diagnosed & adequatley treated PE is 2%
201
Q

What is Hospital-Acquired VTE ?

A

Estimates;
• UK; 60,000 deaths, 32,000 due to hospital admission, 25,000 preventable
• More people die from hospital VTE than breast cancer, HIV &; road traffic accidents combined
• Hospital acquired VTE causes more deaths than hospital acquired infec
• Inflamm state pro-thrombotic- hypercoaguable post op, so when mobilise them cause venous stasis
Facts
• Definition includes any VTE within 90 days of discharge
• Hospital acquired clots- 2/3rds of all VTE
• Registered deaths by VTE in England (2007)- 19,000 (underdiagnosed)
• Autopsy data suggests incidences underestimated

202
Q

What is prevention/treatment of thrombosis?

A

Prophylaxis
• Low doses of anti-coag (e.g. LMWH) during patent judged to be at risk
• Consistent Risk Assessment (work out balance of risk between thrombosis & bleeding) to see if need anticoag
• Appropriate prophylaxis e.g. mechanical prophylaxis may be more appropriate (stockings)
Treatment
• Prompt diagnosis
• Guideline led unified care
NICE guidelines (2010)- reducing risk of VTE in patients admitted to hospital

203
Q

What are TED Anti-Embolism Stockings?

A
  • Mechanical prophylaxis
  • Gradual compression from leg upwards
  • Break in popliteal fossa
  • Reduce DVT incidence by over 50% through peer reviewed studies on over 14,000 patients
  • Useful if can’t take anti-coags
  • Contradindications; peripheral neuropathy, broken skin
  • TED stockings need to be prescribed- need to make sure patient suitable
204
Q

What is Pharmocological Prophylaxis?

A

• Low dose low molecular weight heparin- inducing small amount of anti-coag during time their immobile (&at increased risk)
• E.g. after orthopaedic surgery
• Fondaparinux (synthetic pentasaccharide)
• Newer anticoagulants (can use after major orthopaedic surgery):
o Direct Factor Xa inhibitors: rivaroxaban (apixaban)
o Direct thrombin inhibitors: dabigatran
• Heparin effective in treating venous thrombosis, usually use LMWH.
• Bind to antithrombin enhance its effect (working against F2a &10a).
• All heparins enhance effect of antithrombin
• Unfractionated heparin also anti-2a & 10a, but LMWH predominantly anti-10a effect (less of a dominant effect on coag screening test), but unfractionated heparin prolongs APTT
• Sugar chains in unfractioned heparin longer
• LMWH and unfractioned heparin need anti-thrombin to work

205
Q

How do you diagnose VTE?

A

Exclusion Tests
• Validated numerical clinical probability score: Wells score for DVT &PE (divide into high/ low risk)
• Sensitive quantitative D-dimer with high negative predictive value
• Are screening tests to rule out those most likely not going to have an event
• If high Wells score or D-dimer- need imaging
• Fibrinogen cleaved into monomers fibrin network, F13 coavlently cross links monomers to strengthen polymer
• Plasmin cleaves this netwrok- makes diff patterns
• Can only make D- dimers if have fibrin clot cleaved by plasmin & has been covalently cross linked (know fibrinolysis has started digesting clot)
• D-dimer is a fibrin product of cleavage by plasmin
• So if D-dimer –ve useful in ruling OUT DVT/ PE (not diagnostic)
• Used these in an agreed algorithm

Ultrasound
• Gold standard for DVT
• Next need to get radiological imaging
• Duplex scanning with compression- aid to detect any thrombus
• Highly sensitive & specific for DVT diagnosis
• Look for loss of flow signal, intravascular defects or non collapsing vessels in venous system

Spiral/ Multislice CT Results
• Gold standard for PE
• Can see thrombus in pulmonary artery

VQ Scan Results 2
• Radioactive isotope given- look at ventilation & perfusion
• Looks for mismatch between vantillaiton &perfusion (not directly looking at clot)
• Goof for preg women (breast less exposed to radioactive isotope)

206
Q

What is the treatment for VTE?

A

LMWH for VTE Treatment
• Doses fixed by body weight (or renal function (if have renal impairment))
• Usually once daily by s/c injec
• E.g. enoxaparin 1.5mg/kg s/c od or tinzaparin 175u/kg s/c od
• Treat with LMWH for at least 5 days (treat with warfarin at the same time)
• Overlap with warfarin until INR (international normalised ratio)>2.0 for 2 consecutive days before stop LMW heparin
Novel Anticoagulants
• Rivaroxiban & apixaban both lisenced to use upfront (1 drug given, not 1 durg followed by another like LMWH & warfarin).
• Renal function taken into account
• Are both direct F10a inhibitors (don’t work through anti-thrombin)
• But these drugs don’t have an antidote (so don’t give if patient has high risk of bleeding)
• Warfarin &; heparin have antidotes- can reverse effects
New Oral Anticoagulant with Predictable Pharmacokinetics (No Monitoring)
• Direct thrombin inhibitors- dabigatran
• Direct anti-Xa activity- rivaroxaban, apixaban, endoxaban
Predictable Dose Response
• Warfarin dose varies 40 fold to get target INR- need to monitor
• For NOACs dose uniform for most patients
• No need for routine monitoring: predictable effect
• All agents have dose reduction recommended in specific pops e.g. very elderly, renal impairment for VTE prevention &; AF
• Rapid onset of action- initiation may be done directly with rivaroxaban &; apixaban (no need for LMWH)
• For VTE, recommended dose at initiation greater for rivaroxaban &; apixaban
Management of VTE
• 1st episode of proximal vein DVT or PE: treat for 3-6mnths
• For warfarin: target INR= 2.5
• Recurrrent episodes of VTE: treat with long term anticoagulation
• Proximal DVT or PE in absence of reversible risk factor (e.g. spontaneous)- consider long term anticoagulation
• Recurrent VTE on therapeutic anticoagulation: increase target INR to 3.5 for warfarin (if keep getting recurrent events even if on warfarin)

207
Q

What is Thrombophilia ?

A

• Familial/ acquired disorders of haemostatic mechanism likely to predispose to thrombosis
• Predominantly venous thrombosis when heritable
• Inherited abnormalities must co-segregate in families with clinical thrombosis in the pedigree (need to see evidence of co-segregation)
• Alternative thrombophilia definition- patients who develop VTE (DVT or PE);
o Spontaneously
o Of disproportionate severity (to any factor we can i.d.)
o Recurrently
o At an early age

208
Q

What is Heritable Thrombophilia ?

A

Heritable Thrombophilias (deficiencies of natural anti-coags)
• Antithrombin defieceincy
• Protein C deficiency
• Protein S deficiency
• Activated Protein C resisyance/ FV Leiden
• Dysfibrinogenaemia
• Prothrombin 20210A & FV leiden (polymorphism in the proteins)

209
Q

What is acquired Thrombophilia ?

A

• Antiphospholipid syndrome- autoantibodies against haemostatic system (-vely charge phospholipid)

210
Q

What are the Clinical Features of Thrombophilia ?

A

• DVT
• PE
Superficial thrombophlebitis
• Thrombosis of cerebral, axillary, portal, mesenteric veins
• Arterial thrombosis (APS, only)
• Coumarin induced skin necrosis (PC deficiency)- protein C & S vit K dependant so when start warfarin they fall more rapidly than pro-coag factors so temporarly pro-coag states (worse in patients who are protein C or S deficient- satring with lower of these)
• Obstetric complications: foetal wastage (APS)

211
Q

What is the function of Antithrombin?

A

Is a serene protease inhibitor- against factor 2 a (thrombin) & F10a.

212
Q

What is the function of Protein C &S ?

A
  • Protein C cleaves activated F5 & 8 &inactivates these cofactors (degrades them)- stop pro-coag drive via cofactors
  • Protein S is a cofactor to protein C
213
Q

What is Factor V Leiden?

A
  • Point mutation in pro-coag cofactor 5.
  • Same as activated protein C resistance; have normal amounts of protein C but it doesn’t work properly as an anti-coag in their system as F5 it’s degrading is abnormal (has a point mutation where protein C cleaves it- so resistant to being cleaved by protein C)
  • Genes via founder effect- heterozygous for F5 Leiden survival adv in some way so been populated in Europeans

Factor V Leiden
• APC resistance associated in majority of cases- single point mutation in the Factor V gene : G to A1,691
• Arg 506 is replaced by Gln rendering FV relatively resistant to cleavage by APC
• Most common familial thrombophilia
• 5-8% European population heterozygous
• Not found in Far East and Africa
• Heterozygotes: 3-5 fold OR for venous thrombosis (usually not sole cuase for venous thrombosis- interacts with other risk factors)
• Homozygotes (double copy of factor 5 leiden gene): 30-50 fold OR for venous thrombosis
• Interacts with other risk factors for VTE

214
Q

What is Prothrombin 20210A?f

A
  • Point mutation in 3’ end of untranslated region of prothrombin gene
  • In enhancer region of gene (switches on translation)
  • Weak risk factor on its own- interacts with other defects &; acquired risk factors
  • Associated with increased prothrombin levels
  • 3 fold increase in venous thrombosis
  • Occurs in 1-2% healthy UK population
  • Rare in Asia and Africa
  • Evidence of interaction with other defects
215
Q

What is Antiphospholipid Syndrome ?

A

• Need to have antiphospholipid antibodies (lupus anticoagulant or anticardiolipin antibody) on at least 2 occasions 12 weeks apart in association with :
o Had a venous thrombosis or
o Arterial thrombosis or
o Recurrent fetal loss (>2)
o & reproducible presence of anti-phospholipid antibodies
• May be primary or secondary

216
Q

What is Thrombophilia Screening ?

A
  • Who : Not indicated in unselected patients with VTE; younger patients, positive family history, if management will change
  • When : Avoid confounding factors eg acute VTE, on anticoagulants. Preg &; OCP also affect results
  • Which tests: For selected patients with arterial events or recurrent miscarriage : antiphospholipid antibody screen only
217
Q

What are the causes of anaemia?

A

Causes of Anaemia (other than blood loss)
• Haematinic (nutrients including iron, B12 & folic acid) deficiencies
• 2ndry or chronic disease
• Haemolysis
• Alcohol, drugs, toxins
• Renal impairment (EPO- erythropoietin hormone deficiency)
• Primary haematological;
o Malignant
o Haemoglobin disorders (sickle etc)
o Aplasia
o Congenital

218
Q

What is Megaloblastic Anaemia ?

A
  • Funny appearance of RBCs in bone marrow
  • Characteristic cell morphology caused by impaired DNA synthesis
  • Typical peripheral blood changes- big RBCs (high MCV- macrocytic anaemia);
219
Q

What are Causes of Megalobalstic Change?

A
  • B12 &/or folic acid deficiency
  • Alcohol (puts MCV up & damages liver causing macrocytosis)
  • Drugs- cytotoxics/ folate antagonists/ N2O
  • Heamatological malignancy
  • Congenital rarities (transcobalamin deficiency, orotic aciduria)
220
Q

How do Vit B12 + Folate Cause Anaemia ?

A
  • DNA consists of purine/pyramidine bases- need folate for base synthesis
  • Need B12 for folate generation
  • So low folate or B12 starves DNA of bases
221
Q

What is folic acid?

A

Folic Acid
• Mainly in green veg (folium in spinach), beans, paes, nuts & liver
• Required intake needs decent daily diet
• Absorbed upper small bowel
• 4 months body stores
• (Unlkie B12 don’t need a disease to be deficient in folic acid, if don’t eat it will run out of it’s stores)
Folic Acid Deficiency
• Mainly dietary/ malnutrition
• Malabsorption/ small bowel disease
• Increased usage; preg, haemolysis, inflamm disorders
• Drugs/ alcohol/ ITU

222
Q

What are Features of B12 & Folate Deficiency ?

A

Features Common to B12/ Folate Deficiency
• Can’t tell straight away which one it is
• Megaloblastic anaemia
• Pancytopenia if more severe (deficiency of RBCs, WBCs & platelets)
• Mild jaundice
• Glossitis/ angular stomatitis
• Anorexia/ wt loss
• Sterility

Lab Features of B12 & Folate Deficiency
• Blood count & film (marrow sometimes)
• Bilirubin &; LDH (‘haemolysis’)
• B12 &; folate levels
• Antibodies
• B12 absorption tests +/- IF (use tracers; see where lack of B12 absorption in body- not used anymore as much)
• GI investigations- Crohn’s, malabsorption, blind loop

223
Q

What is Pernicious Anaemia?

A
  • Abs to parietal cells/ intrinsic factor
  • Autoimmune disease
  • Atrophic gastritis with achlorhydria
  • Incidence of Ca stomach
224
Q

What is Subacute Combined Degeneration of the Cord) ?

A
•	Neurological disorder- vit B12 (remember this in back of head) 
•	B12 deficiency can cause neuropathy  
•	Any cause of severe B12 deficiency 
•	Anaemia not an absolute requirement 
•	Demyelination of dorsal &amp; lateral columns 
•	Peripheral nerve damage 
•	Presents as;
o	Peripheral neuropathy/ paraesthesiae
o	Numbness/ distal weakness
o	Unsteady walking 
o	Dementia
225
Q

What is the treatment of pernicious anaemia?

A

• B12 + Folate until B12 deficiency excluded
• B12 x5 then 3 monthly for life for PA (pernicious anaemia)
• Folic acid 5mg daily to build stores
• Need to potassium & iron initially
• Response;
o Retics by day 7 (blood test- reticulocyte count to see response to treatment)
o Blood count/ MCV
o Neuropathy (many months for full effect)

226
Q

What is Haemolysis?

A

• Red cells living less ong- being haemolysed
• Causes;
o Things wrong inside RBC
 Haemoglobinopathy (Sickle cell)
 Enzyme defect (G6PD)
o Things wrong with RBC memb
 Hereditary spherocytosis (round)/ elliptocytosis
o Things wrong external to RBC
 Antibodies (warm/cold)
 Drugs, toxins
 Heart valves (physically destructs red cells- need transfusion)
 Vascualr/ vasculitis/ microangiopathy (abnormal vasculature physically damages RBCs)

227
Q

How do you test for the Presence of Haemolysis?

A
  • Anaemic (or not- compensated)
  • High MCV, macrocytic
  • High reticulocytes
  • Blood film (fragments/ spherocytes)
  • Raised bilirubin, LDH (lactate dehydrogenase- internal red cell enzyme)
  • Low haptoglobins (proteins that bind used Hb- they get used up as too much Hb getting used up)
  • Urinary haemosiderin
228
Q

What is Anaemia of Chronic Disease?

A

• Common general medical issue;
o Malignant/ inflammatory disease/ infectious (often older patients)
o Multiple medical diseases (DM, autoimmune)
• Typically normal MCV- normocytic
• Reduced RBC produc due to; abnormal iron metabolism, poor erythropoietin response & blunted marrow response
• Exclude other anaemias & left with anaemia of chronic disease

• Effects mediated by;
o Release of inflamm cytokines (IL1, IL6, TNF-alpha)- affects iron release form tissue so not available to RBCs & affects bone marrow function
o In particular- hepcidin (regular iron absorption & release from macrophages)

229
Q

What are features of Anaemia of Chronic Disease?

A
  • No other causes of anaemia
  • A suitable medical history
  • Usually mild anaemia, normal MCV
  • Often raised inflamm markers- ESR, CRP, PV etc
  • Normal/high ferritin + low serum iron
  • Normal % saturation transferrin
230
Q

What is the treatment of Anaemia of Chronic Disease?

A
  • The cause if possible
  • Erythropoetin/ iron IV (some respond to erythropoietin, some to iron IV)
  • Transfusion
  • Careful symptom assessment 1st
231
Q

What are the Commonest Causes of Thrombocytopenia?

A
  • Low platelets
  • Drugs, alcohol, toxins
  • ITP (immune thrombocytopenic purpura) (sometimes associated with lymphoma/CLL/ HIV)
  • Other autoimm disease
  • Seen in liver disease & /or hypersplenism (bunch up platelets)
  • Preg (can have normal physiological mild decreased platelets &; range of complications e.g. could be developing a syndrome like pre-eclampsia)
  • Haematological/ marrow diseases
  • Infecs acute or otherwise e.g. acute sepsis/ HIV/ other viral infecs (EBV & many others)
  • Disseminated Intravascular Coag (DIC)- as platelets part of clotting cascade
  • Range of congenital conditions
  • Many others
232
Q

What is Immune Thrombocytopenic Purpura ?

A

• Common- distinguish kids + adults
• Patients present in diff ways- characterised by low platelets
• An immune disorder (may be in association with other autoimm disorders e.g. HIV, lymphomas)
• Occurs on it’s own or as part of;
o Other autoimm disease
o Lymphomas/ CLL
o HIV
• Can be acute one off/ chronic over whole life/ relapsing and remitting with diff levels of severity

233
Q

What is the presentation of Immune Thrombocytopenic Purpura ?

A
Presentation 
•	Bruising or petechiae or bleeding 
•	Platelet count can be anything;
o	<10 urgent/ bleeding 
o	<20 a worry 
o	<30 needs treatment 
o	Otherwise- observe
•	Prodrome/ associated disease?
•	Exclude other causes
234
Q

What is the treatment of Immune Thrombocytopenic Purpura ?

A

Therapy
• Steroids 1st line (immune suppression)
• IV immunoglobulin (saturated spleen with pointless immunoglobulins, so spleen can’t destroy platelets)
• Other immunosuppressives/ splenectomy common next options
• Newer thrombo-mimetics; Eltombopag, Romiplostin (stimulates platelet produc)
• These have thrombopoetin- like properties, getting more widely used

Outcome
•	Usually rapid responses
•	Can relapse after therapy 
•	Rarely life-threatening (haemorrhage)
•	Commonly recurrent (usually in adults than kids)  
•	Some difficult refractory cases
235
Q

What is Thrombotic Thrombocytopenia Purpura ?

A

• Think if someone thrombocytopenic could they have TTP
• Low platelets, usually ill e.g. fevers, usually have anaemia
• Rare but URGENT diagnosis
• Most immune (ADAMTS-13/VWD)
• Suspect if thrombocytopenia &
o Fever
o Neurological symptoms
o Haemolysis (retics/ LDH)
o Seek evidence of microangiopathy (vasculature gone wrong)- blood film shows RBC fragments
Urgent therapy includes;
• Plasma exchange with FFP/ plasma (clears antibodies &replaces ADAMTS-13 enzyme)
• Steroids
• (Vincristine)
• (Rituximab)
• Outcomes vary- can see relapses
• Monitor ADAMTS-13 (enzyme that cleaves VWB factor- in autoimm disease there are antibodies to this)

236
Q

What is Acute Leukaemia ?

A

• Accumulation of early myeloid or lymphoid precursors in bone marrow, blood/ other tissues
• Somatic mutation in a single cell within a population of early progenitor cells
• May arise de novo/ be terminal event of a pre-existing blood disorder (chronic can develop into acute)
• 2 main subgroups;
o Acute myeloid leukaemia (AML)
o Acute lymphoid leukaemia (ALL)
• (Further divided on morphological grounds into various subcategories)
• Leukaemia can affect any of these lineages )in pic on R) so depends on type of leukaemia you have e.g. if in cell erarlier on or later (e.g. erythroblastic leukaemia)- but all similar process

237
Q

What is Acute Myeloblastic Leukaemia ?

A

Blast cells in peripheral blood in AML;
Monomorphic pop (all look the same)
Have holes in nuclei

238
Q

What is Acute Lymphoblastic Leukaemia ?

A

Blast cells in peripheral blood in ALL;

Monomorphic cells, don’t have much cytoplasm (look like nearly all nucleus)

239
Q

What are the features of Acute Leukaemia ?

A
  • Presents with features of bone marrow failure; anaemia, infections, easy bruising & haemorrhage
  • May have all 3 (as acute is rapid onset)
  • Organ infiltration by leukaemia cells may occur e.g. spleen, liver, meninges (need to give drugs directly into spine to get treatment where it’s neede), testes & skin
240
Q

What is the diagnosis of AML?

A
  • Morphology (blood test e.g. after bruising- see cells)
  • Cytochemistry (historical interest staining diff proteins in diff cells)
  • Immunological markers (i.d. antigens on cell surface using monoclonal antibodies from mice marked with fluorescent marker)
  • Cytogenetics, FISH
  • Molecular techniques (PCR)
241
Q

What is the classification of AML?

A
  • FAB classification- ‘morphological’ (how cells looked)
  • WHO classification- ‘risk adapted’ (e.g. group of patients who will do well with therapy, group who need marrow transplant)
242
Q

What is the morphology of AML?

A
  • M1- auer rods (straight lines in cells)= acute myeloid leukaemia
  • M3- promyelocytic anaemia (common exam Q)- has lots of auer rods; cellular contents highly pro-coagulant, so when treat with chemo, cellular contents realised- disrupt clotting forming DIC (get bleeding & blood clots). Now have a new treatment.
  • M6- red cell laeukaemia
243
Q

What are the Immunological Markers of AML?

A
  • Monoclonal antibody determination of surface antigen expression- cornerstone of leukaemia diagnosis
  • Attached to antibody is a coloured marker
  • Immunoflouresence & flouresence activated cell sorting (FACS)- allows rapid leukaemia diagnosis
244
Q

What is the Cytogenetics of AML?

A

Cytogenetics
• Cytogenetic analysis may help confirm diagnosis &; indicate subtype
• Translocations in leukaemia cells- associated with how well patients did with treatment
• Certain abnormalities correlate with prognosis e.g.;
o t(8:21) & t(15:17) in AML (good- do well with treatment)
o Monosomy 7 (patients that had chemo DNA damage develop leukaemia) in AML & t(9:22) in ALL (bad)
Cytogenetic Abnormalities & Prognosis in AML
• Favourable risk group (can just have chemo): t(15:17), t(8:21), inv(16)
• Intermediate risk group (don’t know f will need transplant): (t(9:11), +8, +21
• Poor risk group (will need a bone marrow transplant): t(6:9) inv(3), del(5q), -7, -5
Prognosis: Cytogenetics from MRC AML 10 Trial in Patients <55yrs Old
• CR (complete remission) rate
• Os (overall survival)

245
Q

What are the Immunological Markers of AML?

A

ABNORMALITY
• Numeric change
o ALL- get abnormality of NUMBER of chromosomes (not just translocation)
o High hyperdiploidy (>50 chromosomes) – favourable
o Hyperdiploidy (47-50) – intermediate
o Psuedodiploidy (46+ struc/n.o. change) – intermediate
o Hypodiploidy – poor
• Structural abnormality
o Ph chromosome – very poor
o t(1,19) – poor
o t(4,11) – poor
o t(11,14) – poor
o t(8,14) – very poor
(prognostic significance; favourable, intermediate, poor, very poor)
• Specific oncogenes- poor prognosis

246
Q

What are the Prognostic Factors in ALL ?

A

Poor Prognostic Factors in ALL (host factors which determine how you might do)
• Increasing age (children do very well 90-95% cure rate, young adult in late teens won’t do well)- don’t know why as same disease
• High WCC POOR PROGNOSTIC FACTOR IN ALL (exam Q!!!!!!!!!!!!!!!!)
• Male sex
• Certain cytogentci abnormalities
• Poor response to treatment
• T-ALL & null-ALL

247
Q

What is the Cytogenetic Abnormality of CML/ALL?

A

Cytogenetic Abnormality of CML/ALL: Philadelphia Chromosome
• T(9;22) translocation
• New gene formed
• Fusion onco gene (casues cancer)
• BCR-ABL- codes for tyrosine kinase which causes leukaemia

248
Q

What are the Molecular Abnormalities in Acute Leukaemia ?

A
  • Chromosomal translocations- cause molecular changes (may be important in aetiology)
  • E.g. AML-ETO gene fusion product in t(8;21) translocation (good prognosis) (AML M2)- and monitoring response to treatment & in developing treatment strategies
  • Can use PCR to monitor levels of AML-ETO gene- if rise in PCR test ned more treatment
  • E.g. PML-RARα in t(15;17) translocation, explains response of acute promyelocytic leukaemia (APML- M3) to all trans retinoic acid (ATRA)
  • Give all trans retinoic acid- caused promyelocyte (leukemic cell) to mature to normal cell (so did not get coagulopathy), so if give ATRA before chemo gets over DIC risk
  • AML- ETO gene- way to monitor
  • Retinoic acid recep- way to target
249
Q

What is the Molecular Pathology of AML

A
•	Abnormal cell prolif;
o	FLT3 mutations- affects cell prolif
o	Ras mutations
o	Others: c-KIT mutations
•	Block in differentiation in cells (translocations) ;
o	CBF AML (t(8;21) and inv(16))
o	PML-RARα (t(15;17))
o	MLL translocations (11q23)
•	Tumour suppression;
o	NPM1
•	Patient can have normal chromosomes but still have mutations in genes
250
Q

What is AML with Normal Cytogenetics ?

A
  • Mutations in NPM1 = good (don’t need transplant)
  • Mutations in FLT3 = bad (needs transplant)
  • NPM1+ve/FLT3 ITD-ve gives improved relapse free (61% v 47%) and overall (57% v 23%) survival at 4 years compared with other way round
  • If get combinations in mutations (NPM1 & FLT3) not sure if need transplant or not
251
Q

What is the Significance of Molecular Markers ?

A
  • Risk assessment complex as more than 1 abnormality may be present
  • Current interest is development of molecular ‘signature’ using gene microarray analysis
  • Minimal residual disease monitoring (MRD) using PCR after treatment in some subtypes e.g. APML
  • Some may be a target for therapy e.g. FLT3 inhibitors
252
Q

What is the Management of Acute Leukaemia AML ?

A
  • Induction treatment to obtain remission
  • Then consolidation with further courses of combination chemo (kills abnormal & some normal cells)
  • In younger patients consider bone marrow transplantation (sibling or MUD)
253
Q

What is the Management of Acute Leukaemia ALL?

A
  • All patients receive induction therapy, intensive consolidation chemo &; prophylaxis of meningeal leukaemia with intrathecal methotrexate &; cranial irradiation directly in CNS (to prevent meningeal leukaemia in CNS)
  • Following maintenance chemo is given, or bone marrow transplant in ‘bad-risk’ patients (e.g. with Philadelphia chromosome)
254
Q

What is Neutropenic Sepsis ?

A

Complications of Therapy- Neutropenic Sepsis
• All patients with acute leukaemia receiving intensive chemo will become neutropenic for 10-21 days
• (10 days after treatment will have no neutrophils)
• Neutropenic fever- defined as pyrexia in presence of neutrophil count less than 1.0 x 109/l
• Freq- neutrophil count will fall to levels below 0.2 or unrecordable
Neutropenic Sepsis
• Such patients in grave danger of developing overwhelming gram –ve or gram +ve infec
• Cornerstone of management is immediate administration of broad spectrum IV antibiotics (often Tazocin & Gentamicin) given empirically before results of cultures available
• Chemo- inflamms mucosal surfaces e.g. in bowel- allow infecs
• Other measures (for neuropenic patients) ;
o Prevention
 Protective isolation (protect form risk of infec- air being blown out of rooms)
 Prophylactic antibiotics e.g. levofloxacin (give antibiotic when not infected- risk getting resistant)
 Use granulocyte colony stimulating factors (stim bone marrow- make more neutrophils)
 Strict hand hygiene
o Treatment
 Patient education
 Strict protocols for antimicrobial therapy

255
Q

What is Imatinib?

A

Targeted therapy for CML/ALL ; inhibits tyrosine kinase coded for by BCR-ABL